Download PelleyStep1ReviewInBiochemistry

Survey
yes no Was this document useful for you?
   Thank you for your participation!

* Your assessment is very important for improving the workof artificial intelligence, which forms the content of this project

Document related concepts

Plant nutrition wikipedia , lookup

Metalloprotein wikipedia , lookup

Enzyme wikipedia , lookup

Fatty acid synthesis wikipedia , lookup

Glycolysis wikipedia , lookup

Wilson's disease wikipedia , lookup

Point mutation wikipedia , lookup

Artificial gene synthesis wikipedia , lookup

Metabolism wikipedia , lookup

Amino acid synthesis wikipedia , lookup

Ketosis wikipedia , lookup

Biosynthesis wikipedia , lookup

Biochemistry wikipedia , lookup

Fatty acid metabolism wikipedia , lookup

Transcript
KCUMB Step 1 Review in Biochemistry
John Pelley, PhD
Texas Tech HSC, Lubbock, TX
[email protected]
www.ttuhsc.edu/SOM/success
1.
A 3-year-old African-American male has been diagnosed with malaria. He has become anemic
after starting therapy with primaquine. Laboratory analysis reveals an increase in circulating reticulocytes
and urine analysis reveals hemoglobinuria. Which of the following is the most likely cause of the
hemoglobinuria?
A.
B.
C.
D.
E.
Defect in the β-globin chain
Defect in β-globin chain synthesis
Deficiency of glucose-6-phosphate dehydrogenase (G6PD)
Inhibition of pyruvate kinase
Reduced dietary iron absorption
Correct Response
Option C. G6PD deficiency reduces NADPH production by the pentose phosphate pathway.
NADPH is required to maintain reduced glutathione during oxidative stress from primaquine.
Incorrect Responses
Option A This abnormality is characteristic of several diseases that are typically not caused by the
use of medications, such as sickle cell disease, hemoglobin C disease, and β-thalassemia.
Option B This feature is characteristic of β-thalassemia, an inherited hemolytic disease not caused
by the use of medications.
Option Primaquine does not inhibit pyruvate kinase. However, patients with a genetic deficiency
of this enzyme would experience a hemolytic anemia.
Option E Iron deficiency leads to anemia but not to hemoglobinuria. Also, primaquine does not
affect the absorption of iron from the diet and would not cause iron deficiency anemia.
2.
A 20-year-old male reports painful cramps during exercise. The cramps are usually followed by a
red tint in his urine. Consumption of glucose containing electrolyte replacement drinks before exercising
does not alleviate the symptoms. Additional tests indicate that he has a mild case of hemolytic anemia.
The patient most likely has a deficiency of which of the following enzymes?
A.
B.
C.
D.
E.
Branching enzyme
Glucose-6-phosphatase
α-Glucosidase
Liver phosphorylase
Phosphofructokinase (PFK)
Correct Response
Option E. A deficiency of PFK blocks production of ATP from glycolysis producing painful
cramps during exercise and rhabdomyolysis with concomitant myoglobinuria. It also reduces the half-life
of red blood cells.
Incorrect Responses
Option A A deficiency of branching enzyme, Andersen's disease, produces cirrhosis of the liver
due to abnormal branching of glycogen.
Option B A deficiency of glucose-6-phosphatase, von Gierke's disease, produces massive
hepatomegaly and fasting hypoglycemia within the first few months after birth.
-glucosidase, Pompe's disease, produces cardiomegaly. This
lysosomal disorder is generalized to all tissues, with the heart being the most vulnerable organ.
Option D A deficiency of liver phosphorylase, Hers' disease, produces hepatomegaly, growth
retardation, and fasting hypoglycemia. Symptoms improve with age and typically disappear by puberty.
3.
A 1-year-old boy has liver cirrhosis and hypotonia. His urine is negative for reducing substances.
What enzyme associated with carbohydrate metabolism is most likely causing this child’s condition?
A.
B.
C.
D.
E.
Aldolase B
Galactose-1-phosphate uridyltransferase
Glucose-6-phosphatase
Glucosyl (4:6) transferase
Liver phosphorylase
Correct Response
Option D (glucosyl (4:6) transferase) is correct. This child is exhibiting symptoms of type IV
glycogen storage disease known as Andersen disease, a condition related to a deficiency of the glycogen
branching enzyme glucosyl (4:6) transferase. This deficiency classically causes cirrhosis of the liver or
cardiac tissue damage, usually with death occurring within the first 1 or 2 years of life.
Incorrect Responses
Option A (aldolase B) is incorrect. Deficiency of aldolase B, the enzyme that cleaves fructose 1phosphate, causes hereditary fructose intolerance and leads to fructose (a reducing sugar) in the urine.
Option B (galactose-1-phosphate uridyltransferase) is incorrect. Deficiency of this enzyme, which
is responsible for galactosemia, leads to the presence of galactose (a reducing sugar) in the urine.
Option C (glucose-6-phosphatase) is incorrect. Glucose-6-phosphatase deficiency leads to type I
glycogen storage disease (von Gierke disease). Key features of this condition include a massively
enlarged liver (although usually without cirrhosis), severe hypoglycemia, and failure to thrive.
Option E (liver phosphorylase) is incorrect. Deficiency of liver phosphorylase causes the rare
type VI glycogen storage disease (Hers disease). The presenting symptoms of this type of glycogen
storage disease are similar to those of type I disease, but typically the course of the condition is milder.
4.
A 3-month-old boy has progressive hepatomegaly and mild fasting hypoglycemia. Laboratory
studies show that when the boy is fasting lactic acidosis is absent. A biopsy of liver tissue shows that the
glycogen granules have an abnormal structure. The patient most likely has a deficiency of what enzyme?
A.
B.
C.
D.
E.
Branching enzyme
Debranching enzyme
Glucose-6-phosphatase
α-Glucosidase
Liver phosphorylase
Correct Response
Option B (debranching enzyme) is correct. Debranching enzyme deficiency leads to Cori disease,
which is characterized by hepatomegaly and hypoglycemia. Debranching enzyme is required for the
proper metabolism of glycogen near its branch points. Without it, glycogenolysis ceases when all terminal
chains are at branch points, and no further hydrolysis of glucose residues is possible. The absence of
lactic acidosis implies that pyruvate is not being generated because of the slow operation of the glycolytic
pathway. Treatment of Cori disease, a glycogen storage disease, involves avoidance of fasting and liberal
access to dietary protein for gluconeogenesis.
Incorrect Responses
Option A (branching enzyme) is incorrect. Deficiency of branching enzyme leads to Andersen
disease. The abnormal branching pattern (lack of branches) in the glycogen molecule is believed to result
in cirrhosis of the liver. Affected patients typically present with failure to thrive within the first 18 months
of life and die by 5 years of age.
Option C (glucose-6-phosphatase) is incorrect. Deficiency of glucose-6-phosphatase leads to von
Gierke disease, which affects the liver and kidneys. Children with this enzyme deficiency typically
present at 3 to 4 months of age with massive hepatorenomegaly and fasting hypoglycemia.
-glucosidase leads to Pompe disease, a
lysosomal glycogen storage disease. This condition is generalized to all tissues; the heart is the most
vulnerable organ.
Option E (liver phosphorylase) is incorrect. Liver phosphorylase deficiency leads to Hers disease.
This rare disease initially manifests in childhood as hepatomegaly and fasting hypoglycemia. Both
symptoms improve with age and typically disappear by puberty.
5.
A 2-month-old female infant has coarse facial features, mental retardation, corneal clouding, and
hepatomegaly. Analysis of fibroblasts cultured from the patient most likely shows a deficiency in what
enzyme?
A.
B.
C.
D.
E.
Glucosidase
Hexosaminidase
α-l-Iduronidase
Phosphotransferase
Sphingomyelinase
Correct Response
Option C (α-l-iduronidase) is correct. The infant has the classic features of Hurler syndrome, an
autosomal recessive lysosomal storage disease caused by a deficiency of the enzyme α-l-iduronidase. This
condition, which is classified as a mucopolysaccharidosis, leads to an accumulation of dermatan and
heparan sulfate. Analysis of fibroblast cultures provides evidence for the presence or absence of many
enzymes.
Incorrect Responses
-glucosidase causes Gaucher disease, an
autosomal recessive condition that leads to the accumulation of glucocerebroside in reticuloendothelial
cells (Gaucher cells). This disorder is the most frequent form of the sphingolipidoses and is characterized
by splenomegaly, increased skin pigmentation, and bone lesions.
Option B (hexosaminidase) is incorrect. A deficiency of hexosaminidase A results in the
accumulation of GM2 ganglioside, which is seen in patients with Tay-Sachs disease and Sandhoff
disease. Mental retardation and blindness are associated with both disorders, but they progress more
rapidly in patients with Sandhoff disease.
Option D (phosphotransferase) is incorrect. Loss of a phosphotransferase that tags lysosomal
enzymes for compartmentation in lysosomes causes I-cell disease. Affected individuals have mental
deterioration, skeletal deformities, and elevated lysosomal enzymes in the plasma and urine.
Option E (sphingomyelinase) is incorrect. A deficiency of sphingomyelinase causes NiemannPick disease, an autosomal recessive disorder that leads to the accumulation of sphingomyelin. Signs of
this disorder include an enlarged liver and spleen and mental retardation of rapid onset, usually within the
first 6 months of life (type A disease).
6.
A 25-year-old male medical student who has had diabetes mellitus since the age of 11 is
unconscious when admitted to the emergency department. His wife suspects that he has not been injecting
his insulin on a regular basis. His breath has a fruity odor and his blood glucose is 600 mg/dL. Glucose
and ketone bodies are present in the urine. What substance is most responsible for the patient’s
hyperglycemia?
A.
B.
C.
D.
E.
Cortisol
Epinephrine
Glucagon
Growth hormone
Thyroxine
Correct Response
Option C (glucagon) is correct. Glucagon is the principal hormone that stimulates liver glycogenolysis
and gluconeogenesis, which are primarily responsible for maintaining the hyperglycemic state.
Incorrect Responses
Option A (cortisol) is incorrect. Cortisol functions in the liver to block insulin action, but this patient has
not been taking his insulin.
Option B (epinephrine) is incorrect. Epinephrine acts primarily on adipose tissue (free fatty acids) and
skeletal muscle (glycogen) to mobilize energy. Neither of these tissues releases glucose.
Option D (growth hormone) is incorrect. Growth hormone is not appreciably increased in the untreated
diabetic.
Option E (thyroxine) is incorrect. Thyroxine primarily functions in controlling metabolic rate and in
regulating growth and development and is not altered in diabetic ketoacidosis.
7.
A 5-year-old male has had psychomotor retardation and a chronic lactic acidosis since birth. His
fasting blood glucose is 100 mg/dL and he is not anemic. Which of the following is most likely deficient
in this patient?
A
Phosphoenolpyruvate carboxykinase
B.
Pyruvate carboxylase
C.
Pyruvate dehydrogenase
D.
Pyruvate kinase
Correct Response
Option C. The patient most likely has a deficiency of pyruvate dehydrogenase resulting in an
accumulation of pyruvate with concomitant formation of lactate and reduced ATP production leading to
nervous system dysfunction.
Incorrect Responses
Option A A deficiency of phosphoenolpyruvate carboxykinase, a gluconeogenic enzyme, results in
hypoglycemia; this patient has a normal fasting blood glucose.
Option B A deficiency of pyruvate carboxylase produces lactic acidosis, but it also causes fasting
hypoglycemia and is frequently fatal by 6 months.
Option D A deficiency of pyruvate kinase produces a hemolytic anemia; this patient has a normal
hematocrit. Lactic acidosis is not a characteristic of this enzyme deficiency.
8.
A 3-year-old male is brought to the pediatrician with failure-to-thrive. Physical examination of
the patient reveals hepatomegaly. The fasting blood glucose is 30 mg/dL and no increase in the fasting
blood glucose is seen with a glucagon challenge test is performed. The patient most likely has a
deficiency of which of the following enzymes?
A
Aldolase B
B.
Fructokinase
C.
Galactokinase
D.
Galactose-1-phosphate uridyltransferase
E.
Glucose-6-phosphatase
Correct Response
Option E The patient has von Gierke’s disease which produces a fasting hypoglycemia. Accumulation of
glycogen leads to hepatomegaly. Glucagon cannot increase in the fasting blood glucose.
Incorrect Responses
Option A A deficiency of aldolase B produces hereditary fructose intolerance. Fructose ingestion
produces failure to thrive, hepatomegaly, and hypoglycemia, but the liver would show a normal response
to glucagon.
Option B A deficiency of fructokinase produces a benign, asymptomatic disease. The only symptom is an
accumulation of fructose in the blood and urine.
Option C Galactokinase deficiency produces cataracts due to an increased conversion of galactose to
galactitol in the lens of the eye. Fasting blood glucose would be normal.
Option D Deficiency of galactose-1-phosphate uridyltransferase produces classic galactosemia with poor
weight gain, hepatosplenomegaly, cataracts, and vomiting. Fasting blood glucose would be normal.
9.
An 18-year-old male has a history of episodes of jaundice since birth. Physical examination
shows conjunctival pallor and splenomegaly. Blood analysis reveals slightly increased indirect bilirubin,
no elevation of homocysteine, and normocytic anemia with increased peripheral blood reticulocytes.
Numerous spiculated red blood cells (RBCs) are apparent in the peripheral smear. The patient most likely
has impaired activity for which of the following enzymes?
A
Homocysteine methyltransferase
B.
Glucose-6-phosphate dehydrogenase (G6PD)
C.
Pyruvate dehydrogenase
D.
Pyruvate kinase
Correct Response
Option D Pyruvate kinase deficiency leads to a decrease in cellular ATP. Dehydration causes cells to
appear spiculated and splenic removal of damaged cells increases unconjugated bilirubin.
Incorrect Responses
Incorrect Responses
Option A The activity of homocysteine methyltransferase is impaired in either a folate or a B12
deficiency. This produces elevated blood homocysteine and a megaloblastic anemia instead of the
normocytic anemia seen here.
Option B G6PD deficiency leads to a deficiency of glutathione in RBCs producing anemia due to oxidant
damage. RBCs do not show spiculation (due to dehydration) and splenomegaly is not a feature of the
disease.
Option C Pyruvate dehydrogenase deficiency does not cause a hemolytic anemia. Pyruvate
dehydrogenase is a mitochondrial enzyme and RBCs do not contain mitochondria.
10.
A 25-year-old male of Greek descent complains of prolonged fatigue after a recent illness. Blood
analysis shows normocytic anemia and urine analysis shows hemoglobinuria. Which of the following is
the most likely cause of this patient’s anemia?
A
decreased glutathione peroxidase activity
B.
iron deficiency
C.
folate deficiency
D.
reduced concentrations of ATP
E.
reduced concentrations of NADH
Correct Response
Option A Glutathione peroxidase activity is limited by reduced glutathione availability. This patient has a
glucose 6-phosphate dehydrogenase deficiency and cannot maintain adequate reduced glutathione
concentrations.
Incorrect Responses
Option B Iron deficiency would cause an anemia but not hemoglobinuria.
Option C Folate deficiency would cause megaloblastic anemia without hemoglobinuria.
Option D A pyruvate kinase deficiency would lead to energy deficient RBCs that are removed by the
spleen. Splenomegaly and jaundice with spiculated RBCs are characteristic of this anemia.
Option E NADH concentrations are already maintained at low concentrations. Instead, reduced
concentrations of NADPH underlie the symptoms in this patient.
11.
A 33-year-old female alcoholic complains of tiredness and disorientation approximately 6 hours
after a cocktail party. Her blood glucose is at 25 mg/dL. Which of the following would you expect to find
elevated in her bloodstream?
A
Ethanol
B.
Lactic acid
C.
Chylomicrons
D.
Acetate
Correct Response
Option B The metabolism of ethanol produces an excess of NADH that pushes pyruvate toward lactate
removing it as a substrate for gluconeogenesis leading to a drop in blood glucose.
Incorrect Responses
Option A Ethanol would have been metabolised by the liver within 6 hours.
Option C Chylomicron remnants would have been removed from the circulation by the liver within 6
hours.
Option D Acetate produced from the oxidation of ethanol is further metabolized in the liver to acetylCoA. It would not build up and spill into the blood.
12.
A 7 year old male was brought to the clinic by his mother with the complaint that he was having
episodes of acting and talking “as if he were drunk.” These episodes occurred after stressful periods such
as a febrile illness or fatigue. On examination the boy showed rapid breathing and rapid heart rate.
Laboratory analysis showed elevated blood lactate and blood pyruvate, although the lactate to pyruvate
ratio was normal. Blood pH was slightly acid at 7.30 (normal 7.35-7.45). Blood glucose and ketones were
normal. No ragged red fibers were observed in a skeletal muscle biopsy. What is the most likely
diagnosis?
A
B.
C.
D.
hypoxia
pyruvate carboxylase deficiency
mitochondrial disease
pyruvate dehydrogenase deficiency
Correct Response
Option D Pyruvate dehydrogenase deficiency results in the simultaneous appearance of lactate and
pyruvate and the neurological symptoms observed.
Incorrect Responses
Option A Hypoxia would only elevate lactate. The simultaneous appearance of lactate and pyruvate
indicate an inability to metabolize pyruvate.
Option B Pyruvate carboxylase deficiency would be accompanied by hypoglycemia (the blood glucose
was normal).
Option C Mitochondrial disease is ruled out since ragged red fibers are absent
13.
A 6 day-old female full term infant was brought to the emergency room with an enlarged liver,
jaundice, failure-to-thrive, and urinary excretion of reducing sugars and albumin. Blood analysis showed
a normal hemoglobin level and an elevated serum AST and ALT. Urinalysis with glucose oxidase was
negative. She tolerated feeding with sucrose, maltose, glucose, and fructose at doses of 2 g/kg, but she
vomited after lactose feeding. What other tissues will be affected by this disease?
A
B.
C.
D.
E.
Red blood cells
spleen
Heart muscle
Lens of the eye
CNS
Correct Response
Option D The reducing sugar with negative glucose oxidase indicates that this patient has galactosemia, a
disease that produces cataracts.
Incorrect Responses
Option A The reducing sugar with negative glucose oxidase indicates that this patient has galactosemia, a
disease that does not affect red blood cells.
Option B The reducing sugar with negative glucose oxidase indicates that this patient has galactosemia, a
disease that does not increase the activity of the spleen.
Option C The reducing sugar with negative glucose oxidase indicates that this patient has galactosemia, a
disease that does not affect the heart.
Option E The reducing sugar with negative glucose oxidase indicates that this patient has galactosemia, a
disease that does not affect CNS function.
14.
A 56-year-old male complains of chest pain associated with shortness of breath and fatigue. A
coronary angiogram indicates 85% blockage of the left anterior descending coronary artery. Which of the
following pumps is most directly affected by this myocardial ischemia?
A
Calcium
B.
bicarbonate
C.
Potassium
D.
Proton
E.
Sodium
Correct Response
Option D Reduced blood flow to the heart slows mitochondrial electron transport and the proton pumps
that are coupled to it since oxygen is the terminal electron acceptor of the electron-transport chain. This
leads to a drop in the supply of ATP in the myocardium.
Incorrect Responses
Option A Reduced availability of oxygen most directly affects the proton pumps coupled to the electron
transport chain. Eventually the decreasing supply of ATP affects the other active ion pumps such as the
calcium pumps.
Option B Reduced availability of oxygen most directly affects the proton pumps coupled to the electron
transport chain. Bicarbonate is transported in exchange for chloride and does not require energy.
Option C Reduced availability of oxygen most directly affects the proton pumps coupled to the electron
transport chain. Eventually the decreasing supply of ATP affects the other active ion pumps such as the
potassium pumps.
Option E Reduced availability of oxygen most directly affects the proton pumps coupled to the electron
transport chain. Eventually the decreasing supply of ATP affects the other active ion pumps such as the
sodium pumps.
15.
A 25-year-old male is brought to the emergency room after falling from a ladder. He has
sustained a ruptured spleen and is going into hypovolemic shock as a result of blood loss. He is conscious
and his respiratory rate is elevated. Which of the following biochemical processes is most likely increased
in this patient?
A
anion gap
B.
electron transport chain
C.
Na+,K+ATPase
D.
citric acid cycle activity
E.
oxidative phosphorylation
Correct Response
Option A Anaerobic metabolism in the hypoxic peripheral tissues increases the production of lactic acid
leading to a lactic acidosis anion gap.
Incorrect Responses
Option B Anaerobic metabolism in the hypoxic peripheral tissues reduces the amount of oxygen as a
terminal electron acceptor and slows the electron transport chain.
Option C Anaerobic metabolism in the hypoxic peripheral tissues results in decreased synthesis of ATP
leading to a decrease in the ATP-dependent Na /K+ pump.
Option D Anaerobic metabolism in the hypoxic peripheral tissues leads to an increase in NADH which
allosterically inhibits isocitrate dehydrogenase, the rate controlling step in the citric acid cycle.
Option E Anaerobic metabolism in the hypoxic peripheral tissues slows the flow of electrons through the
electron transport chain which is tightly coupled to oxidative phosphorylation.
16.
A 5 year old male is examined by his pediatrician for fatigue, nausea, and lightheadedness
brought on when meals are skipped. Physical examination reveals hepatomegaly. Laboratory results
indicate lactic acidosis, hyperlipidemia, elevated serum uric acid, and marked hypoglycemia.
Administration of fructose did not result in conversion to glucose. What other organs would be expected
to be enlarged?
A
kidney
B.
spleen
C.
heart
D.
brain
E.
skeletal muscle
Correct Response
Option A When the G6P concentrations increase due to the absence of the glucose 6-phosphatase enzyme,
glycogen synthase is stimulated to synthesize additional glycogen and enlarging the kidney.
Incorrect Responses
Option B The spleen is not enlarged in Von Gierke’s disease.
Option C Von Gierke’s disease is not characterized by heart enlargement. Heart muscle already lacks the
glucose 6-phosphatase enzyme.
Option D The brain does not store significant glycogen and would consume any excess glucose, thus
preventing excess glycogen storage.
Option E Skeletal muscle already lacks the glucose 6-phosphatase enzyme.
17.
A 59 year old male complained during an annual physical exam of aching in his thighs during his
5 mile morning runs. No significant changes were made in the length or speed of the running schedule.
He has a history of elevated blood lipids and has been taking a standard daily dose of a statin drug for the
past two years. A routine laboratory profile was normal, but the creatine phosphokinase (CPK) value was
elevated. What deficiency is created by the statin drug that is causing the symptoms?
A
isoprene
B.
cholesterol
C.
dehydroepiandrosterone
D.
vitamin D
Correct Response
Option A Ten isoprene units are needed to form the tail on Coenzyme Q10. A deficiency in isoprene in
the cholesterol pathway would create a coenzyme Q10 deficiency and impair energy metabolism in
muscle.
Incorrect Responses
Option B Statins lower cholesterol but do not create a deficiency.
Option C The reduced synthesis of cholesterol with statin therapy is not sufficient to limit precursors for
steroid hormone synthesis.
Option D The reduced synthesis of cholesterol with statin therapy is not sufficient to limit precursors for
vitamin D synthesis.
18.
A 52-year-old man with chronic alcoholism has a plasma lipid profile that contains an increase in
very-low-density lipoprotein (VLDL). The mechanism that best explains the increase in VLDL in both
the blood and liver in chronic alcoholics is
A.
B.
impaired function of apolipoprotein B-100
inactivation of capillary lipoprotein lipase
C.
D.
increased β-oxidation of fatty acids
increased synthesis of glycerol 3-phosphate
Correct Response
Option D (increased synthesis of glycerol 3-phosphate) is correct. Ethanol is metabolized in the
liver and eventually forms acetate, with NADH as another major product. Chronic consumption of
ethanol causes a shift in the cytosolic balance between NAD+ and NADH in favor of NADH. The altered
NAD+/NADH ratio causes less effective operation of the citric acid cycle, halts efficient oxidation of
fatty acids, and causes a shift from dihydroxyacetone phosphate to glyceraldehyde 3-phosphate and
eventually to glycerol 3-phosphate. The abundance of glycerol and free fatty acids results in an increased
production of triacylglycerol, with increased export from the liver as VLDL, and fatty liver. These
conditions are often seen in chronic alcoholics.
Incorrect Responses
Option A (impaired function of apolipoprotein B-100) is incorrect. Alcohol consumption alone
does not cause impairment of the function of any of the apolipoproteins. However, this impairment, which
occurs in abetalipoproteinemia, does result in fatty liver.
Option B (inactivation of capillary lipoprotein lipase) is incorrect. Alcohol consumption has not
been shown to inhibit the activity of peripheral tissue lipoprotein lipase. Such inhibition leads to an
increase in triacylglycerol levels in the plasma; it is a stimulus for hepatic production of VLDL.
-oxidation of fatty acids) is incorrect. The reducing environment in the cell
from the excess NADH+ leads to a decrease in the oxidation of fatty acids, as stated in the discussion of
Option D.
19.
A 3-month-old boy has rapidly progressive central nervous system (CNS) deterioration,
spasticity, and failure to thrive. Physical examination is significant for marked hepatosplenomegaly. Liver
biopsy shows an increase of sphingomyelin in the lysosomes. What enzyme is most likely deficient in this
infant?
A.
B.
C.
D.
Arylsulfatase A
Hexosaminidase A
Liver phosphorylase
Sphingomyelinase
Correct Response
Option D (sphingomyelinase) is correct. A deficiency of sphingomyelinase causes Niemann-Pick
disease, an autosomal recessive disease that results in the accumulation of sphingomyelin in the
lysosomes. Signs of the disorder include an enlarged liver and spleen and mental retardation of rapid
onset, usually within the first 6 months of life.
Incorrect Responses
Option A (arylsulfatase A) is incorrect. A deficiency of arylsulfatase A, which results in the
accumulation of a sulfate-containing ceramide in the lysosomes, causes metachromatic leukodystrophy,
an autosomal recessive disease. Mental retardation and demyelination are typical in patients with this
deficiency.
Option B (hexosaminidase A) is incorrect. A deficiency of hexosaminidase A results in the
accumulation of GM2 ganglioside, which is seen in patients with Tay-Sachs disease and Sandhoff
disease. Mental retardation and blindness are associated with both disorders, but they progress more
rapidly in patients with Sandhoff disease.
Option C (liver phosphorylase) is incorrect. A deficiency in liver phosphorylase causes Hers
disease, an autosomal recessive disease that results in the inability of the liver to mobilize glucose from
glycogen. Hepatomegaly is present, but there are no symptoms of CNS deterioration or failure to thrive,
nor is sphingomyelin increased in lysosomes.
20.
A 25-year-old woman sees her physician because of exhaustion and diarrhea. Physical
examination shows that she has low blood pressure, and blood studies show that she is anemic. Other
studies show that she has a tumor of the pituitary gland that could produce hypofunction. Which of the
following metabolic conversions is most likely affected by this pituitary tumor?
A.
B.
C.
D.
E.
cortisol from cholic acid
7-dehydrocholesterol from cholesterol
mevalonate from HMG CoA
pregnenolone from cholesterol
progesterone from pregnenolone
Correct Response
Option D (pregnenolone from cholesterol) is correct. The patient has signs of pituitary
hypofunction (exhaustion, diarrhea, low blood pressure, and anemia) and is diagnosed with a tumor of the
pituitary gland, in the sella turcica. Formation of pregnenolone from cholesterol is the rate-limiting step in
the synthesis of steroid compounds. It is regulated by the action of ACTH on the cytochrome P450 sidechain cleavage enzyme.
Incorrect Responses
Option A (cortisol from cholic acid) is incorrect. Cholic acid, a bile salt, is not found in the
steroid pathway.
Option B (7-dehydrocholesterol from cholesterol) is incorrect. The sterol 7-dehydrocholesterol is
an intermediate substance in the conversion of lanosterol to cholesterol. It accumulates in the skin as a
precursor to cholecalciferol (vitamin D3).
Option C (mevalonate from HMG CoA) is incorrect. HMG-CoA reductase is the rate-limiting
step in cholesterol synthesis, not steroid hormone synthesis.
Option E (progesterone from pregnenolone) is incorrect. Formation of progesterone from
pregnenolone is not directly affected by loss of ACTH.
21.
A 3-year-old girl who is listless about 4 hours after eating is found to have a deficiency of
carnitine. What substrate is most likely the child’s primary energy source?
A.
B.
C.
D.
E.
Branched-chain amino acids
Fatty acids
Fructose
Glucose
Ketone bodies
Correct Response
Option D (glucose) is correct. This child is lethargic, although the mobilization of free fatty acids
from adipose tissue and -oxidation in the liver is occurring. The lack of carnitine causes the carnitine
shuttle, which transports fatty acids from the cytosol to the mitochondrial matrix, to be inoperative.
Therefore, 12 ATP derived from acetyl CoA, 3 ATP from NADH, and 2 ATP from FADH2 are not
available. Therefore, the child is subsisting primarily on glucose derived from glycogenolysis as the
primary source of energy for all tissues. Gluconeogenesis is suppressed because it relies on fatty acid
catabolism as an energy source. This condition produces hypoglycemia. Treatment often involves fatty
acids that contain less than 12 carbons, which can pass directly into the mitochondria without the
carnitine shuttle.
Incorrect Responses
Option A (branched-chain amino acids) is incorrect. The metabolism of branched-chain amino
acids supplies succinyl CoA for gluconeogenesis. However, without the ATP supplied by fatty acid
oxidation, gluconeogenesis cannot operate optimally.
Option B (fatty acids) is incorrect. Without carnitine, fatty acids with more than 12 carbons
cannot enter the mitochondria to be oxidized to acetyl CoA; therefore, they provide no energy.
Option C (fructose) is incorrect. The body does not store fructose in appreciable quantities to
supply energy during fasting conditions.
Option E (ketone bodies) is incorrect. Ketone bodies are produced from large excesses of acetyl
CoA (i.e., from oxidation of fatty acids). Because carnitine is not available to shuttle fatty acids into the
mitochondrion, no ketone bodies are produced in the liver.
22.
A 22-year-old woman with poorly controlled gestational diabetes delivers a large-for-gestational
age infant at 24 weeks. Within 6 hours after birth, the newborn has respiratory difficulties associated with
tachypnea and cyanosis. What substance is most likely deficient in this newborn?
A.
B.
C.
D.
E.
Cardiolipin
Ceramide
Dipalmitoyl phosphatidylcholine
Ganglioside
Sphingomyelin
Correct Response
Option C (dipalmitoyl phosphatidylcholine) is correct. The newborn has respiratory distress
syndrome (RDS), which is caused by a lack of production of lung surfactant by type II pneumocytes in
the lungs. Dipalmitoyl phosphatidylcholine (lecithin), the primary lung surfactant, reduces surface
tension, preventing the collapse of alveoli. RDS is very common in premature infants (usually <32 weeks'
gestation) because of lung immaturity. Additionally, RDS frequently occurs in infants born to diabetic
mothers as the result of fetal hyperglycemia and hyperinsulinemia, which delay surfactant production.
Incorrect Responses
Option A (cardiolipin) is incorrect. Cardiolipins are lipids that occur in high concentration in the
inner mitochondrial membrane.
Option B (ceramide) is incorrect. Ceramide, which is a precursor to sphingomyelin and
ganglioside, occurs primarily in the myelin sheath.
Option D (ganglioside) is incorrect. Gangliosides are cerebrosides that occur in myelin.
Option E (sphingomyelin) is incorrect. Sphingomyelins occur in nerve tissue and blood.
23.
A 3-year-old child has ambiguous genitalia, diffuse pigmentation, and hypertension. Laboratory
studies show an increase in plasma adrenocorticotropic hormone, a decrease in serum cortisol, and an
increase in urine 17-ketosteroids. Chromosome analysis shows an XX genotype. What enzyme involved
in adrenal steroid synthesis is most likely deficient in this patient?
A.
B.
C.
D.
11β-Hydroxylase
17α-Hydroxylase
18-Hydroxylase
21-Hydroxylase
Correct Response
Option A (11β-hydroxylase) is correct. This child has adrenogenital syndrome, which consists of
a group of autosomal recessive conditions that involve enzyme deficiencies in adrenal steroid synthesis,
leading to hypocortisolism. A deficiency of 11β-hydroxylase results in an increase in steroid compounds
proximal to the enzyme block and a decrease in those compounds distal to the block. Levels of the 17ketosteroids dehydroepiandrosterone and androstenedione are increased because they are proximal to the
block. Androstenedione is converted to testosterone, which is converted to dihydrotestosterone by 5αreductase. Dihydrotestosterone is primarily responsible for producing ambiguous genitalia in females. 11Deoxycortisol is increased, because it is proximal to the enzyme block, and cortisol is decreased, because
it is distal to the block. There is also an increase in 11-deoxycorticosterone, a weak mineralocorticoid that
is responsible for salt retention, leading to hypertension.
Incorrect Responses
-Hydroxylase is responsible for the conversion of
pregnenolone to 17-hydroxypregnenolone and progesterone to 17-hydroxyprogesterone. Each compound
is again converted to 17-ketosteroids. Because the 17-ketosteroi
hydroxylase deficiency is not a possible cause of the symptoms.
Option C (18-hydroxylase) is incorrect. 18-Hydroxylase, which converts corticosterone to
aldosterone, does not cause hypocortisolism or any of the symptoms evident in this patient.
Option D (21-hydroxylase) is incorrect. In 21-hydroxylase deficiency, the 17-ketosteroids are
increased, but the 17-hydroxycorticoids and the mineralocorticoid compounds (11-deoxycorticosterone,
corticosterone, and aldosterone) are decreased. Mineralocorticoid deficiency leads to the loss of sodium in
the urine and retention of potassium in the blood. Sodium loss results in hypotension, not hypertension.
24.
A 1-year-old child has hypertension, diffuse pigmentation, and female external genitalia. A
chromosome analysis shows an XY genotype. The plasma adrenocorticotropic hormone concentration is
increased, serum cortisol is decreased, and a random urine potassium is increased. What enzyme involved
in adrenal steroid synthesis is most likely deficient in this patient?
A.
B.
C.
D.
11 -Hydroxylase
17 -Hydroxylase
18-Hydroxylase
21-Hydroxylase
Correct Response
Option B (17 -hydroxylase) is correct. The child has adrenogenital syndrome, which consists of
a group of autosomal recessive conditions that involve enzyme deficiencies in adrenal steroid synthesis,
leading to hypocortisolism. 17 -Hydroxylase converts pregnenolone and progesterone to 17 hydroxypregnenolone and 17-hydroxyprogesterone, respectively, which are further converted to the
ketosteroids dehydroepiandrosterone and androstenedione. Androstenedione is converted to
dihydrotestosterone, a strong androgen. Without the stimulation of dihydrotestosterone, the external
genitalia in a male fetus has the appearance of a female. Therefore, this patient, who has an XY genotype
and external female genitalia, is a male pseudohermaphrodite. Cortisol is decreased because it is distal to
the block. Hypocortisolism causes an increase in adrenocorticotropic hormone (loss of negative
feedback), resulting in excessive stimulation of the adrenal cortex (bilateral hyperplasia) and an
accumulation of compounds proximal to the enzyme block. Because 17 -hydroxylase is not involved in
mineralocorticoid synthesis, 11-deoxycorticosterone, corticosterone, and aldosterone are all increased,
leading to sodium retention and hypertension.
Incorrect Responses
-hydroxylase leads to an increase
in 11-deoxycortisol (a 17-hydroxycorticoid); 17-ketosteroids; testosterone; dihydrotestosterone
(ambiguous genitalia in females, precocious puberty in males); and 11-deoxycorticosterone, a weak
mineralocorticoid that leads to salt retention and hypertension.
Option C (18-hydroxylase) is incorrect. 18-Hydroxylase converts corticosterone to aldosterone.
Absence of 18-hydroxylase would not cause hypocortisolism or any of the symptoms evident in this
patient.
Option D (21-hydroxylase) is incorrect. A deficiency of 21-hydroxylase in females leads to
ambiguous genitalia (female pseudohermaphroditism) and to precocious puberty in males. The 17hydroxycorticoids (11-deoxycortisol and cortisol) and all mineralocorticoids are decreased, because they
are distal to the enzyme block. Loss of mineralocorticoid activity leads to sodium loss in the urine,
volume depletion, and hypotension.
25.
A 23-year-old woman has just eaten a meal consisting mainly of carbohydrates. What best
describes the activity of capillary lipoprotein lipase and hormone-sensitive lipase in the adipose tissue of
this individual?
A.
B.
C.
D.
Decreased capillary lipoprotein lipase
Decreased capillary lipoprotein lipase
Increased capillary lipoprotein lipase
Increased capillary lipoprotein lipase
Decreased hormone-sensitive lipase
Increased hormone-sensitive lipase
Decreased hormone-sensitive lipase
Increased hormone-sensitive lipase
Correct Response
Option C (increased capillary lipoprotein lipase; decreased hormone-sensitive lipase) is correct.
In the well-fed state, insulin stimulates the synthesis of capillary lipoprotein lipase, and apolipoprotein CII activates the enzyme. These actions result in hydrolysis of circulating chylomicrons derived from the
diet and very-low-density lipoprotein synthesized in the liver, causing the release of fatty acids and
monoglycerides. Insulin inactivates hormone-sensitive lipase in adipose tissue by activating phosphatase,
which dephosphorylates the enzyme.
Incorrect Responses
Option A (both capillary lipoprotein lipase and hormone-sensitive lipase are decreased) and
Option D (both capillary lipoprotein lipase and hormone-sensitive lipase are increased) are incorrect.
Neither type of lipase in the adipose tissue of this individual is decreased nor increased at the same time,
because capillary lipoprotein lipase is associated with fat storage and hormone-sensitive lipase is
associated with fat mobilization.
Option B (decreased capillary lipoprotein lipase and increased hormone-sensitive lipase) is
incorrect. In the fasting state, capillary lipoprotein lipase activity is decreased, whereas hormone-sensitive
lipase is increased via activation by epinephrine and absence of insulin. This releases fatty acids and
glycerol into the circulation.
26.
A 75-year-old man develops urinary retention. Findings on physical examination include an
enlarged, soft prostate gland and a bladder that is distended to the height of the umbilicus. A renal
ultrasound shows enlargement of the bladder, both ureters, and the calyceal system of both kidneys. The
hormone responsible for the enlarged prostate is most likely produced by what enzyme?
A.
B.
C.
D.
E.
Aromatase
17α-Hydroxylase
21-Hydroxylase
Oxidoreductase
5α-Reductase
Correct Response
Option E (5α-reductase) is correct. The enzyme 5α-reductase converts testosterone to
dihydrotestosterone, the hormone that is mainly responsible for benign prostatic hyperplasia (BPH) and
prostate cancer. The patient has BPH, which is the most common cause of urinary retention with
hydronephrosis (distention of the kidney) in elderly men.
Incorrect Responses
Option A (aromatase) is incorrect. Aromatase irreversibly converts androgens (e.g.,
dihydrotestosterone, testosterone, androstenedione) to estrogens (e.g., estradiol, estrone). Aromatase does
not convert androgens to other types of androgens.
-Hydroxylase is important in the conversion of 17hydroxypregnenolone to dehydroepiandrosterone and 17-hydroxyprogesterone to androstenedione.
Production of these two weak androgens (dehydroepiandrosterone and androstenedione), which are also
known as 17-ketosteroids, are produced in both the adrenal cortex and Leydig cells in the testes. These
hormones are not involved in the pathogenesis of BPH.
Option C (21-hydroxylase) is incorrect. This enzyme is important in the conversion of 17hydroxyprogesterone to 11-deoxycortisol in steroid synthesis in the adrenal cortex. These compounds are
not involved in the pathogenesis of BPH.
Option D (oxidoreductase) is incorrect. This enzyme converts androstenedione to testosterone in
the adrenal cortex and the Leydig cells. These hormones are not involved in the pathogenesis of BPH.
27.
A 2-year-old child suffers from a rare genetic disease that is characterized by fat malabsorption,
spinocerebellar degeneration, hemolytic anemia, and pigmented retinopathy. Total cholesterol and
triacylglycerol levels are extremely low, and levels of chylomicrons, very low-density lipoprotein
(VLDL), and low-density lipoprotein (LDL) are not detectable. The patient most likely has an
abnormality associated with what apolipoprotein?
A.
B.
C.
D.
Apolipoprotein A (apoA)
Apolipoprotein B (apoB)
Apolipoprotein C-II (apoC-II)
Apolipoprotein E (apoE)
Correct Response
Option B (apolipoprotein B) is correct. The patient has abetalipoproteinemia, a rare autosomal
recessive disease characterized by the inability to form chylomicrons or VLDL. ApoB is necessary for the
formation of chylomicrons (apoB-48), VLDL (apoB-100), and LDL (apoB-100). The absence of
chylomicrons, which carry diet-derived triacylglycerol (TG), and VLDL, which carries endogenously
synthesized TG, accounts for the markedly decreased levels of TG in the blood. Furthermore, LDL is the
major carrier for cholesterol and is derived from hydrolysis of VLDL, so serum cholesterol levels are also
very low. The clinical findings in this patient (like most patients with abetalipoproteinemia) result from
vitamin E deficiency. This fat-soluble vitamin is normally reabsorbed with fat. In abetalipoproteinemia,
TG and the fat-soluble vitamins cannot be packaged into chylomicrons in the small intestine. TG
accumulates in the lamina propria of the villi and prevents the reabsorption of fat and fat-soluble vitamins
from the intestinal lumen, which leads to steatorrhea (excess fat in stools). Vitamin E deficiency is
associated with spinocerebellar degeneration and hemolytic anemia. Treatment involves megadoses of
vitamin E.
Incorrect Responses
Option A (apolipoprotein A) is incorrect. ApoA is unique to high-density lipoprotein (HDL).
Deficiency of apoA, a rare disease, is associated with the absence of HDL and a predisposition for
premature coronary artery disease.
Option C (apolipoprotein C-II) is incorrect. ApoC-II is important in the activation of capillary
lipoprotein lipase, which hydrolyzes TG and VLDL to release fatty acids and glycerol. ApoC-II is
deficient in familial hyperchylomicronemia syndrome (type I hyperlipoproteinemia) and is associated
with an increase in chylomicrons that begins in early childhood.
Option D (apolipoprotein E) is incorrect. Apo E is present on chylomicrons, VLDL, and
intermediate-density lipoproteins (IDL). A mutant form of apoE characterizes familial
dysbetalipoproteinemia (type III hyperlipoproteinemia). The apoB and apoE receptors in the liver cannot
remove chylomicron remnants and IDL that have the mutant form of apoE, so these remnants accumulate
in the blood and increase TG and cholesterol levels.
28.
A 16-year-old boy has bilateral Achilles tendon xanthomas. His father died of an acute
myocardial infarction at 32 years of age. What biochemical defect is most likely responsible for the
physical finding in this patient?
A.
B.
C.
D.
E.
Decreased clearance of very-low-density lipoprotein (VLDL)
Deficient levels of apolipoprotein C-II
Deficient levels of apolipoprotein E
Deficient levels of capillary lipoprotein lipase
Deficient levels of low-density-lipoprotein (LDL) receptors
Correct Response
Option E (deficient levels of low-density-lipoprotein receptors) is correct. Achilles tendon xanthomas are
pathognomonic for familial hypercholesterolemia (type II hyperlipoproteinemia), which is inherited in an
autosomal dominant fashion. Absent or defective LDL receptors lead to an increase in the level of serum
cholesterol, which deposits in the tendon sheaths. Patients die at an early age of an acute myocardial
infarction or stroke.
Incorrect Responses
Option A (decreased clearance of very-low-density lipoprotein) is incorrect. Decreased clearance of
VLDL is called type IV hyperlipoproteinemia. Only plasma levels of triacylglycerol are primarily
increased.
Option B (deficient levels of apolipoprotein C-II) is incorrect. Apolipoprotein C-II activates capillary
lipoprotein lipase in the fed state. Capillary lipoprotein lipase hydrolyzes triacylglycerol in chylomicrons
and VLDL, which causes the release of glycerol and fatty acids into the peripheral circulation. Deficiency
of apolipoprotein C-II is rare and occurs mainly in children (as type I hyperlipoproteinemia). In this
condition, chylomicron levels are markedly elevated, and hence serum triacylglycerol levels are
increased.
Option C (deficient levels of apolipoprotein E) is incorrect. Apolipoprotein E primarily mediates the
uptake of chylomicrons and intermediate-density lipoproteins by hepatocytes. Deficiency of
apolipoprotein E is manifested phenotypically as type III hyperlipoproteinemia, also known as
dysbetalipoproteinemia or remnant disease, because levels of chylomicron remnants and intermediatedensity lipoproteins are increased. Both serum cholesterol and triacylglycerol levels are increased.
Option D (deficient levels of capillary lipoprotein lipase) is incorrect. Genetic absence of this enzyme is a
rare childhood disease (type I hyperlipoproteinemia) and is characterized by an increase in chylomicron
levels, similar to the situation in apolipoprotein C-II deficiency.
29.
A 48-year-old man is scheduled for a lipid profile. He is instructed to fast for 12 hours before the
blood sample is drawn at 8 am the next day. Lipid analysis shows:
Serum cholesterol: 250 mg/dL
Serum high-density lipoprotein cholesterol: 25 mg/dL
Serum triacylglycerol: 300 mg/dL
A refrigerated plasma sample shows a creamy layer on the surface of the plasma, but the plasma itself is
clear. What change best explains the appearance of the refrigerated sample?
A.
B.
C.
D.
E.
Increased level of chylomicrons
Increased level of chylomicrons and low-density lipoproteins (LDL)
Increased level of chylomicrons and very-low-density lipoproteins (VLDL)
Increased level of LDL
Increased level of VLDL
Correct Response
Option A (increased level of chylomicrons) is correct. Chylomicrons consist of mainly diet-derived
triacylglycerol. An increase in triacylglycerol produces turbidity in plasma. Because chylomicrons are the
least dense of the lipoprotein fractions, they normally float on top of plasma (supranate) after the sample
is refrigerated. The turbid supranate in the patient’s sample indicates a high level of chylomicrons. This
result suggests that the patient had not fasted, but instead probably ate a breakfast containing saturated
fats before the blood sample was obtained.
Incorrect Responses
Option B (increased level of chylomicrons and LDL) and Option D (increased level of LDL) are
incorrect. The level of chylomicrons is increased in this patient (see discussion for Option A). LDL
carries cholesterol, but an increase in cholesterol does not produce turbidity.
Option C (increased level of chylomicrons and VLDL) and Option E (increased level of VLDL) are
incorrect. The chylomicron level is increased in this patient. VLDL is the primary vehicle for carrying
endogenously synthesized triacylglycerol. Because VLDL is slightly denser than is the chylomicron
fraction, an increased level of VLDL would cause the entire tube of plasma to be turbid and there would
be no supranate. The plasma in this patient’s sample is clear, however, which indicates that the VLDL
concentration is not increased.
30.
A 25-year-old female has just finished eating a lunch consisting of a ham sandwich, potato salad,
orange juice, and vanilla ice cream. Which of the following is most likely to be increased?
A
adenylate cyclase activity
B.
protein kinase A activity
C.
hormone-sensitive lipase activity
D.
lipoprotein lipase activity
Correct Response
Option D Insulin which is increased by consumption of carbohydrates leads to an increase in lipoprotein
lipase activity on the endothelial surface to break down triglycerides in chylomicrons.
Incorrect Responses
Option B (increased level of chylomicrons and LDL) and Option D (increased level of LDL) are
incorrect. The level of chylomicrons is increased in this patient (see discussion for Option A). LDL
carries cholesterol, but an increase in cholesterol does not produce turbidity.
Option C (increased level of chylomicrons and VLDL) and Option E (increased level of VLDL) are
incorrect. The chylomicron level is increased in this patient. VLDL is the primary vehicle for carrying
endogenously synthesized triacylglycerol. Because VLDL is slightly denser than is the chylomicron
fraction, an increased level of VLDL would cause the entire tube of plasma to be turbid and there would
be no supranate. The plasma in this patient’s sample is clear, however, which indicates that the VLDL
concentration is not increased.
31.
A 43-year-old man with chronic renal failure, who requires hemodialysis, complains of
generalized bone pain. Physical examination shows bowing of the legs and a sallow complexion. When
the man’s blood pressure is taken, his thumb flexes into his palm. A radiograph of the bones shows
generalized osteopenia. The patient most likely has a deficiency of which of the following vitamins?
A
Niacin
B.
ascorbic acid
C.
Vitamin A
D.
Vitamin D
E.
Vitamin K
Correct Response
Option D Chronic renal failure can render a patient unable to form 1,25-dihydroxycholecalciferol
producing hypocalcemia and signs of tetany. Calcium is mobilized from bone producing osteopenia.
Incorrect Responses
Option A Niacin deficiency (pellagra) is characterized by diarrhea, dermatitis, and dementia and is not
directly involved in bone or calcium metabolism.
Option B Ascorbic acid is involved in the proper formation of collagen and a deficiency would lead to
fragile capillaries due to abnormal connective tissue, but not to osteopenia.
Option C Vitamin A toxicity can produce pain in the long bones, but it does not have a role in bone
maintenance or calcium homeostasis.
Option E Vitamin K deficiency is not directly involved in bone resorption or calcium homeostasis.
32.
A 2-month-old male is brought to the pediatrician to treat a worsening cough. Physical
examination reveals a diaper rash and oral thrush. Radiography reveals a chest infiltrate consistent with
Pneumocystis carinii pneumonia. Blood analysis shows a complete lack of lymphocytes. Which of the
following enzymes is most likely deficient in this infant
A
B.
C.
D.
Adenosine deaminase
Hypoxanthine-guanine phosphoribosyltransferase (HGPRT)
Orotate phosphoribosyl transferase
Xanthine oxidase
Correct Response
Option A Severe combined immunodeficiency disease (SCID) is a result of a deficiency of the enzyme
adenosine deaminase. SCID is characterized by decreased production of both B- and T-cell precursors.
Incorrect Responses
Option B HGPRT deficiency produces Lesch-Nyhan syndrome which is characterized by mental
retardation and self-mutilation.
Option C Orotate phosphoribosyl transferase deficiency produces orotic aciduria
Option D Xanthine oxidase deficiency produces hypouricemia and increased excretion of
xanthine and hypoxanthine.
33.
A neonatal nurse notices that the urine produced by a newborn infant has the odor of maple syrup.
What biochemical reaction is most likely defective?
A.
Metabolism of aromatic amino acids
B.
Metabolism of branched-chain amino acids
C.
Metabolism of sulfur-containing amino acids
D.
One-carbon transfer reactions
E.
Transformation of carbohydrates to amino acids
Correct Response
Option B (metabolism of branched-chain amino acids) is correct. A defect in the oxidation of the
branched-chain amino acids valine, leucine, and isoleucine (all of which are essential amino acids) leads
to a build-ketoacids. The accumulation of these acids in the urine leads to its
pathognomonic maple-syrup odor.
Incorrect Responses
Option A (metabolism of aromatic amino acids) is incorrect. The aromatic amino acids
phenylalanine, tyrosine, and tryptophan, which are primarily increased in chronic liver disease, contribute
to the mental status abnormalities associated with hepatic encephalopathy. Branched-chain amino acids
are used to reduce the effect of the aromatic amino acids in patients with chronic liver disease.
Option C (metabolism of sulfur-containing amino acids) is incorrect. This action would be more
likely with homocystinuria, which is caused by a deficiency of cystathionine synthetase. Homocystinuria
is characterized by mental retardation, osteoporosis, and lens dislocation.
Option D (one-carbon transfer reactions) is incorrect. This is seen most often in cases of folate
deficiency. Tetrahydrofolate transfers methylene groups (–CH2–) to serine to produce 5,10-methylene
tetrahydrofolate, which in turn transfers the methylene group to deoxyuridine monophosphate to produce
deoxythymidine monophosphate for DNA synthesis.
Option E (transformation of carbohydrates to amino acids) is incorrect. The urine of individuals
with this wide variety of deficiencies does not have the characteristic odor of maple syrup.
34.
A healthy adult is placed on a diet that lacks tyrosine, but is otherwise balanced. As a result of
this diet, nitrogen balance is most likely to
A.
be achieved
B.
become negative
C.
become positive
D.
progress from a negative state to equilibrium
E.
progress from a positive state to equilibrium
Correct Response
Option A (be achieved) is correct. A state of nitrogen balance exists when the amount of nitrogen
excreted is equal to the amount ingested. This is seen in healthy adults whose intake of dietary protein is
adequate. Tyrosine, a nonessential amino acid, is synthesized from phenylalanine in the diet. Failure to
include tyrosine in the diet would not affect the nitrogen balance in this individual, who should
experience no adverse effects from the diet.
Incorrect Responses
Option B (become negative) is incorrect. A negative nitrogen balance exists when more nitrogen
is excreted than is ingested. This occurs when the protein intake is insufficient or of low quality (e.g., the
diet does not provide the correct amounts of all of the essential amino acids) or during catabolic states.
Option C (become positive) is incorrect. A positive nitrogen balance exists when more nitrogen is
ingested than is excreted. This occurs primarily in anabolic states (e.g., during a "growth spurt").
Option D (progress from a negative state to equilibrium) and Option E (progress from a positive
state to equilibrium) are incorrect. Transitional stages of nitrogen balance are not common (whether the
end state is positive or negative), because the conditions that lead to a nitrogen imbalance are chronic in
nature. For example, a negative nitrogen balance is common following surgery, during advanced stages of
cancer, or in individuals with starvation syndromes (e.g., kwashiorkor).
35.
A 42-year-old man is taking isoniazid to treat tuberculosis. He is most at risk of developing a
deficiency of what amino acid?
A.
Arginine
B.
Aspartate
C.
Glutamate
D.
Glutamine
E.
Glycine
Correct Response
Option B (aspartate) is correct. Patients who take isoniazid are at risk for developing pyridoxine
(vitamin B6) deficiency. Pyridoxine is required for the proper functioning of the transaminases, the
-amino acids. Aspartate is produced from
transamination of oxaloacetate by aspartate aminotransferase (AST).
Incorrect Responses
Option A (arginine) is incorrect. Synthesis of arginine, an essential amino acid, also occurs during
the urea cycle and does not require pyridoxine.
Option C (glutamate) is incorrect. Although glutamate can be produced from the reverse action of
the transaminase
-ketoglutarate by glutamate dehydrogenase, which
does not require pyridoxine.
Option D (glutamine) is incorrect. Glutamine is synthesized from glutamate by glutamine
synthetase, which does not require pyridoxine.
Option E (glycine) is incorrect. Glycine is synthesized from serine by a hydroxymethyl
transferase, which does not require pyridoxine.
36.
A 48-year-old man with cirrhosis of the liver is not oriented to time or place. After he is placed on
a low-protein diet, he experiences some improvement in mental status. What enzyme is most likely
responsible for these abnormalities in mental status?
A.
Arginase
B.
Argininosuccinate lyase
C.
Argininosuccinate synthetase
D.
Carbamoyl phosphate synthetase I
E.
Ornithine carbamoyltransferase
Correct Response
Option D (carbamoyl phosphate synthetase I) is correct. This patient is suffering from hepatic
encephalopathy, which is most likely induced by a high-protein diet. The excess nitrogen waste from
protein is usually handled by the liver (urea cycle), which is damaged in this patient. The ammonia from
protein degradation by urease-producing organisms in the bowel is converted to urea in the liver via the
urea cycle. The rate-limiting step is the synthesis of carbamoyl phosphate from NH3 and CO2, which is
catalyzed by carbamoyl phosphate synthetase I. Because the urea cycle is dysfunctional, blood NH3
levels increase and blood urea nitrogen decreases. A low-protein diet decreases the amount of NH3 that
the urea cycle must metabolize.
Incorrect Responses
Option A (arginase) is incorrect. Arginase helps cleave arginine to produce ornithine and urea.
Option B (argininosuccinate lyase) is incorrect. Argininosuccinate lyase catalyzes the conversion
of argininosuccinate to arginine and fumarate.
Option C (argininosuccinate synthetase) is incorrect. Argininosuccinate synthetase catalyzes the
condensation of aspartate and citrulline to form argininosuccinate.
Option E (ornithine carbamoyltransferase) is incorrect. Ornithine transcarbamoylase catalyzes the
condensation of ornithine and carbamoyl phosphate to form citrulline.
37.
A 1-year-old child with mental retardation has a deficiency of dihydrobiopterin reductase. This
enzyme deficiency is most closely related to what disorder?
A.
Galactosemia
B.
Maple syrup urine disease
C.
Niemann-Pick disease
D.
Phenylketonuria (PKU)
E.
Wilson disease
Correct Response
Option D (phenylketonuria) is correct. The secondary form of PKU results from an inability to
regenerate tetrahydrobiopterin. This lack of tetrahydrobiopterin affects not only the conversion of
phenylalanine to tyrosine, but also the hydroxylation of tyrosine and tryptophan, leading to deficiencies of
neurotransmitters and additional central nervous system effects. Dietary restriction of phenylalanine is not
sufficient to reverse the neurologic effects. Control of the primary form of PKU, which is caused by
phenylalanine hydroxylase deficiency, involves dietary restriction of phenylalanine and tyrosine
supplementation.
Incorrect Responses
Option A (galactosemia) is incorrect. Galactosemia, which may present as mental retardation, is
caused by a deficiency in galactose-1-phosphate uridylyl transferase. The enzyme does not require
tetrahydrobiopterin as a cofactor.
Option B (maple syrup urine disease) is incorrect. Maple syrup urine disease is caused by a
deficiency of branched-keto acid dehydrogenase. This enzyme does not require
tetrahydrobiopterin as a cofactor.
Option C (Niemann-Pick disease) is incorrect. Niemann-Pick disease also presents as mental
retardation with hepatosplenomegaly. It is caused by a defect in the lysosomal hydrolytic enzyme
sphingomyelinase. This enzyme does not require tetrahydrobiopterin as a cofactor.
Option E (Wilson disease) is incorrect. Wilson disease is caused by a defect in the excretion of
copper into the bile. Excess copper in the liver leads to chronic liver disease and decreased synthesis of
ceruloplasmin, a copper-binding protein. Excess copper in the blood leads to deposits in the eye (KayserFleischer rings) and the lenticular nuclei in the brain, resulting in movement disorders. Ceruloplasmin
does not require tetrahydrobiopterin as a cofactor.
38.
A 42-year-old man who works in a battery factory complains of fatigue, headaches, abdominal
pain, and chronic diarrhea. Laboratory studies show a hemoglobin of 10 g/dL and a mean corpuscular
coarse basophilic stippling. What is an additional expected laboratory finding?
A.
Decreased serum ferritin
B.
Decreased zinc RBC protoporphyrin
C.
Increased hemoglobin A2 (HbA2 ) and hemoglobin F (HbF)
D.
-aminolevulinic acid (ALA)
Correct Response
Option D (increased urinary -aminolevulinic acid) is correct. The patient has lead poisoning,
which commonly occurs in battery factory employees. Elevation of ALA is usually a clue that lead
poisoning is present, because lead denatures ALA dehydratase, which converts ALA to porphobilinogen.
Increased levels of ALA, along with lead, produce demyelination in the brain and increased vessel
permeability leading to cerebral edema. Lead poisoning often results in a microcytic anemia, with
characteristic coarse basophilic stippling of RBCs because of denaturation of ribonuclease, the enzyme
that degrades ribosomes.
Incorrect Responses
Options A (decreased serum ferritin) is incorrect. Serum ferritin, a circulating soluble protein
storage form of iron, correlates with iron stores in the bone marrow macrophages. It is elevated in lead
poisoning, where iron accumulates in the mitochondria in developing nucleated RBCs and produces
ringed sideroblasts, which contain iron-laden mitochondria surrounding the nucleus. When the
sideroblasts die in bone marrow, the iron is stored in macrophages, leading to an increase in iron stores in
the marrow as well as iron in the blood.
Option B (decreased zinc RBC protoporphyrin) is incorrect. In lead poisoning, denaturation of
ALA dehydratase and ferrochelatase reduces the amount of zinc protoporphyrin in the RBCs that is
available to combine with iron to form heme. In the past, zinc RBC protoporphyrin levels were used to
screen for lead poisoning. However, these values are also increased in other conditions (e.g., iron
deficiency), which limits the usefulness of the test. Currently, the blood lead level is used as the screening
and confirmatory test for lead poisoning.
Option C (increased hemoglobin A2 and hemoglobin F) is incorrect. Lead denatures enzymes
involved in hemoglobin synthesis (e.g., ALA dehydratase and ferrochelatase) and in the degradation of
-thalassemia
minor is characterized by an increase in HbA2 and HbF. I
-globin
39.
A 1-year-old male child is brought to the pediatrician because his parents have noticed that the
boy is mutilating himself by chewing on his lips. On examination, the child is found to be mentally
retarded and manifesting signs of gouty arthritis. What enzyme involved in nucleoside metabolism is
most likely deficient in this child?
A.
Adenosine deaminase
B.
Hypoxanthine-guanine phosphoribosyltransferase (HGPRT)
C.
Phosphoribosylpyrophosphate synthase
D.
Thymidine kinase
E.
Xanthine oxidase
Correct Response
Option B (hypoxanthine-guanine phosphoribosyltransferase) is correct. This child is manifesting
signs and symptoms of Lesch-Nyhan syndrome, an X-linked recessive disorder. A complete deficiency of
HGPRT causes this condition. HGPRT is responsible for the salvage of purines by converting
hypoxanthine and guanine to their monophosphated forms. Lack of this enzyme leads to destruction of
free hypoxanthine and guanine, increased uric acid levels, and ensuing mental retardation and, less
importantly, gout. The cause of the self-mutilating behavior in Lesch-Nyhan syndrome is unknown.
Incorrect Responses
Option A (adenosine deaminase) is incorrect. Adenosine deaminase catalyzes the conversion of
adenosine to inosine in the degradation pathway of adenosine. A deficiency of this enzyme leads to a
form of severe combined immunodeficiency disease (SCID).
Option C (phosphoribosylpyrophosphate synthase) is incorrect. Phosphoribosylpyrophosphate
synthase catalyzes the rate-limiting step in the synthesis of purine nucleotides and plays an important role
in many other pathways. The most common alteration of this synthase is overactivity, which leads to an
overproduction of purines and ensuing hyperuricemia and gout.
Option D (thymidine kinase) is incorrect. Thymidine kinase is an enzyme in the salvage pathway
for pyrimidines (specifically thymine) that converts thymidine to TMP. Drugs, such as acyclovir, which
are used primarily to treat herpes, inhibit viral versions of this enzyme.
Option E (xanthine oxidase) is incorrect. Xanthine oxidase, the terminal enzyme in the
degradation pathway of purines, converts xanthine to uric acid. This enzyme can be inhibited by
allopurinol, which reduces the production of uric acid and abates the occurrences of gout in individuals
prone to attacks.
40.
A 1-year-old boy has a history of recurrent bacterial and fungal infections. Serum protein
-globulin region. Further testing shows increased concentrations of dATP
in white blood cells. What is the most likely diagnosis?
A.
Adenosine deaminase deficiency
B.
Bruton agammaglobulinemia
C.
Lesch-Nyhan syndrome
D.
Oroticaciduria
E.
Purine nucleoside phosphorylase deficiency
Correct Response
Option A (adenosine deaminase deficiency) is correct. Adenosine deaminase deficiency leads to
an extremely large accumulation of dATP, which inhibits ribonucleotide reductase. The resulting
impairment in both T-cell (recurrent fungal infections) and B-cell function (hypogammaglobulinemia
-globulin region) has led to the name severe combined immunodeficiency disease or SCID.
Incorrect Responses
Option B (Bruton agammaglobulinemia) is incorrect. Bruton agammaglobulinemia is an X-linked
recessive disorder in which pre-B cells cannot mature into B cells, causing an isolated
hypogammaglobulinemia.
Option C (Lesch-Nyhan syndrome) is incorrect. Lesch-Nyhan syndrome results from a genetic
deficiency in hypoxanthine-guanine phosphoribosyltransferase, which prevents the salvage of both
hypoxanthine and guanine. The blocked salvage simultaneously leads to a decrease in the concentrations
of inosine monophosphate and guanosine monophosphate (feedback inhibitors of the first step in purine
synthesis) and an increase in phosoribosylpyrophosphate (the limiting substrate in the first step of purine
synthesis).
Option D (oroticaciduria) is incorrect. Oroticaciduria results from a deficiency of the enzymes
-monophosphate. The buildup of orotic acid leads to abnormal growth
and megaloblastic anemia. Treatment with a uridine-rich diet is successful.
Option E (purine nucleoside phosphorylase deficiency) is incorrect. Purine nucleoside
phosphorylase deficiency results in an accumulation of dGTP and dATP, both of which inhibit
ribonucleotide reductase. This causes impairment of T-cell (but not B-cell) function.
41.
A full-term male infant begins to show irritability and lethargy 36 hours after birth. Laboratory
studies show a plasma ammonia level that is 60 times normal and a urinary orotate level that is 100 times
normal. What enzyme is most likely defective in this infant?
A.
Arginase
B.
Carbamoyl phosphate synthetase II
C.
Ornithine transcarbamoylase
D.
Serine hydroxymethyltransferase
Correct Response
Option C (ornithine transcarbamoylase) is correct. Defects of the urea cycle retard the disposal of
free NH3, causing it to accumulate in the bloodstream. The more proximal the enzyme defect in the urea
cycle, the more pronounced the increase in NH3 in the blood. A deficiency of ornithine
transcarbamoylase in the mitochondrion blocks the entry of nitrogen into the urea cycle as carbamoyl
phosphate by preventing its conversion to citrulline. The carbamoyl phosphate in the mitochondrion
eventually leaks into the cytoplasm, where it accelerates the pyrimidine pathway, leading to the
appearance of orotic acid in the urine.
Incorrect Responses
Option A (arginase) is incorrect. Arginase is responsible for cleaving arginine to urea and
ornithine; a deficiency of arginase leads to a buildup of arginine. Typically, the metabolism of arginine by
other pathways (creatine and nitric acid synthase) or its elimination in the urine prevents this
accumulation. In rare instances of excessive protein intake, a deficiency of arginase may lead to a mild-tomoderate hyperammonemia.
Option B (carbamoyl phosphate synthetase II) is incorrect. Carbamoyl phosphate synthetase II,
the cytosolic form of the enzyme, catalyzes one of the initial steps in pyrimidine synthesis. If this enzyme
were deficient, the urea cycle, which would have a functional carbamoyl phosphate synthetase I, would
remove any accumulated ammonia.
Option D (serine hydroxymethyltransferase) is incorrect. Serine hydroxymethyltransferase
reversibly transfers a single-carbon unit from methylene tetrahydrofolate to glycine to synthesize serine.
A deficiency of this enzyme does not result in hyperammonemia.
42.
A 4-day-old infant has physiologic jaundice of the newborn. She receives phototherapy with blue
light to reduce the level of bilirubin in her blood and eliminate jaundice in the skin. What mechanism best
explains the manner in which phototherapy reduces jaundice?
A.
Increases conjugating enzymes in the liver
B.
Increases synthesis of vitamin D in the skin
C.
Metabolizes bilirubin to biliverdin
D.
Oxidizes bilirubin to a water-soluble form
Correct Response
Option D (oxidizes bilirubin to a water-soluble form) is correct. Water-soluble forms of bilirubin
are nontoxic and are excreted in the urine. Phototherapy with blue light causes bilirubin to undergo photooxidation, which produces the water-soluble form.
Incorrect Responses
Option A (increases conjugating enzymes in the liver) is incorrect. Phototherapy does not
increase the synthesis of conjugating enzymes in the liver.
Option B (increases synthesis of vitamin D in the skin) is incorrect. Ultraviolet light increases
vitamin D synthesis in the skin. Vitamin D does not alter the concentration of bilirubin in the skin or
blood.
Option C (metabolizes bilirubin to biliverdin) is incorrect. Biliverdin, which is more watersoluble than bilirubin, is converted to bilirubin in macrophages before it is conjugated in the liver.
Conversion of biliverdin to bilirubin is an energy-requiring reaction and is not reversed by phototherapy.
43.
A 28-year-old woman has an intermittent history of severe abdominal pain that typically occurs
after she has consumed a few glasses of wine or after she has been on "crash diets." In visits to many
different physicians over the past few years, she has undergone several laboratory tests (e.g., complete
blood cell count, electrolytes), barium studies, and laparoscopic examinations, none of which has revealed
the cause of her pain. Her physician now orders a urine specimen for culture. The woman collects the
specimen and leaves it near a window for about 30 minutes, when she notices that the urine color has
changed from pale yellow to wine-red. What compound is most likely responsible for the color change?
A.
Hemoglobin
B.
Myoglobin
C.
Porphobilin
D.
Urobilin
E.
Uroporphyrin I
Correct Response
Option C (porphobilin) is correct. Porphobilinogen, which is excreted in the urine, is colorless,
but in the presence of light, it is oxidized to porphobilin, which has a wine-red color. The recurrent,
neurologically induced abdominal pain precipitated by either drugs that induce the liver cytochrome P450
system or dietary restriction is characteristic of acute intermittent porphyria, an autosomal dominant
disease caused by a deficiency of uroporphyrinogen I synthase. This enzyme catalyzes the conversion of
porphobilinogen to hydroxymethylbilane, one of the reactions in heme synthesis. Deficiency of the
enzyme leads to a proximal accumulation of both porphobilinogen and aminolevulinic acid. Confirmation
of the diagnosis involves documentation of the enzyme deficiency in red blood cells. Treatment involves
carbohyd
-aminolevulinic acid synthase, the
rate-limiting enzyme in heme synthesis.
Incorrect Responses
Option A (hemoglobin) is incorrect. The presence of hemoglobin in the urine, or hemoglobinuria,
occurs primarily with intravascular hemolysis. Hemoglobinuria is characterized by red-colored urine that
does not change in color after exposure to light.
Option B (myoglobin) is incorrect. The presence of myoglobin, or myoglobinuria, occurs
primarily after strenuous exercise, leading to rhabdomyolysis (rupture of muscle). Myoglobinuria is
characterized by red-colored urine that does not change in color after exposure to light.
Option D (urobilin) is incorrect. Urobilin, the oxidation product of urobilinogen, is the pigment
normally responsible for the yellow color of urine. This substance normally occurs in trace amounts in
urine. An increase in urobilin turns the urine dark yellow. It does not turn a wine-red color upon exposure
to light.
Option E (uroporphyrin I) is incorrect. Levels of uroporphyrin I are increased in porphyria
cutanea tarda, a disorder characterized by a deficiency of uroporphyrinogen decarboxylase. The presence
of uroporphyrin I in urine colors it wine-red when voided. Porphyria cutanea tarda is associated with
photosensitive skin lesions, such as vesicles and bullae, fragile skin, and fine hair on the face.
44.
A 5-year-old black child is brought to a medical missionary hospital in Africa. The child is
apathetic, and physical examination shows reddish brown hair, a protuberant abdomen, hepatomegaly by
percussion, ascites, and pitting edema of the lower extremities. What feature is most likely responsible for
the hepatomegaly in this patient?
A.
Decreased albumin content of the blood
B.
Dec
-oxidation of fatty acids
C.
Decreased synthesis of apolipoproteins
D.
Increased intake of fatty food
E.
Increased synthesis of fatty acids
Correct Response
Option C (decreased synthesis of apolipoproteins) is correct. The patient has kwashiorkor, which
is a result of a decreased protein intake despite a normal total calorie intake. A diet consisting primarily of
carbohydrates results in an increase in glycerol-3-phosphate, an intermediate in glycolysis. Glycerol is the
carbohydrate backbone of triacylglycerol, which, in turn, is packaged into very-low-density lipoprotein
(VLDL) in hepatocytes. Proteins are water soluble, so circulating lipoproteins, including VLDL, are
protected by a covering that includes apolipoproteins. In kwashiorkor, apolipoprotein synthesis is
decreased as a result of the low protein intake. Thus, triglyceride accumulates in the hepatocytes, with the
accumulation producing fatty changes in the liver and massive hepatomegaly. The reddish-colored hair
found in patients with kwashiorkor is a result of copper deficiency (flag sign); the ascites and pitting
edema are related to hypoalbuminemia, which leads to a decrease in plasma oncotic pressure and leakage
of a transudate into interstitial tissue and body cavities.
Incorrect Responses
Option A (decreased albumin content of the blood) is incorrect. Hypoalbuminemia is caused by
decreased protein intake, but it is not the cause of the fatty changes in the liver. It is directly responsible
for the ascites and pitting edema in the patient.
-oxidation of fatty acids) and Option E (increased synthesis of fatty acids) are
-oxidation of fatty acids provide
more fatty acids for synthesis of triacylglycerol, these mechanisms are not the primary ones for the fatty
liver changes seen in kwashiorkor. They assume a more prominent role in producing the fatty liver
associated with excessive alcohol intake.
Option D (increased intake of fatty food) is incorrect. Patients with kwashiorkor eat primarily
carbohydrates. An increase in consumption of fatty food does not lead to a fatty liver, because VLDL
derives from carbohydrates, not saturated fats.
45.
A 53-year-old male with chronic myelogenous leukemia has begun treatment with multiple
antineoplastic agents. He soon develops renal failure. Urinalysis shows an acid pH and the presence of
crystals that have become deposited in the renal tubules. What treatment will most likely help reduce the
deposition of the crystals in the renal tubules?
A. increased fluids
B. allopurinol
C. ciprofloxacin
D. lithotripsy
E. hydrochlorothiazides
Correct Response
Option B (allopurinol) is correct. The crystals in this patient are uric acid produced from
degradation of dead cancer cells. This complication is prevented by allopurinol, which prevents the
conversion of xanthine to uric acid.
Incorrect Responses
Option A (increased fluids) is incorrect. Renal stones are often treated with increased fluids that
allow the stones to pass naturally. The crystals in this patient are uric acid produced from degradation of
dead cancer cells and are trapped in renal tubules making this therapy ineffective.
Option C (ciprofloxacin) is incorrect. The crystals in this patient are not because of a bacterial
infection. They are uric acid produced from degradation of dead cancer cells.
Option D (lithotripsy) is incorrect. Lithotripsy breaks renal stones down into smaller particles
that can be passed. The uric acid crystals in this patient are already small particles, but they are trapped in
the renal tubules.
Option E (hydrochlorothiazides) is incorrect. Hydrochlorothiazide is prescribed for calcium
containing renal stones. The crystals in this patient are uric acid produced from degradation of dead
cancer cells.
46.
A 38-year-old male has been diagnosed with tuberculosis. He is treated with isoniazid and
rifampin. He develops a microcytic anemia associated with a high serum ferritin concentration. A bone
marrow examination shows increased iron stores and numerous ringed sideroblasts. What enzyme is most
likely affected by the chemotherapy?
A.
Glucose-6-phosphate dehydrogenase
B.
Methylmalonyl CoA mutase
C.
Glutathione reductase
D.
D-aminolevulinic acid synthase
E.
Methemoglobin reductase
Correct Response
Option D (d-aminolevulinic acid synthase) is correct. Isoniazid is a competitive inhibitor for
pyridoxine and prevents its action as a cofactor for delta-aminolevulinic acid synthase. This reduces the
amount of protoporphyrin available to form heme.
Incorrect Responses
Option A (Glucose-6-phosphate dehydrogenase) is incorrect. Inhibition of glucose 6-phosphate
dehydrogenase would produce a normocytic type of hemolytic anemia, but it is not affected by isoniazid
or rifampin.
Option B (Methylmalonyl CoA mutase) is incorrect. Inhibition of methylmalonyl CoA mutase
would produce a megaloblastic anemia, but it is not affected by isoniazid or rifampin.
Option C (glutathione reductase) is incorrect. Inhibition of glutathione reductase would lead to
hemolytic anemia, but it is not affected by isoniazid or rifampin.
Option E (Methemoglobin reductase) is incorrect. Inhibition of methemoglobin reductase would
not produce anemia, but would stimulate red cell synthesis. It is not affected by isoniazid or rifampin.
A 42-year-old male who works at a firing range complains of headaches, loss of sensation, weakness, a
metallic taste in the mouth, and uncoordinated walking. Laboratory studies show a hemoglobin of 9 g/dL
and a mean corpuscular volume (MCV) of 70 mm3. His red blood cells (RBCs) are hypochromic with
coarse basophilic stippling. What is an additional expected laboratory finding?
A.
Decreased serum ferritin
B.
Decreased zinc RBC protoporphyrin
C.
Increased hemoglobin A2 (HbA2) and hemoglobin F (HbF)
D.
Increased urinary d-aminolevulinic acid (ALA)
Correct Response
Option D (Increased ALA) is incorrect. Elevation of ALA is usually a clue that lead poisoning is
present, because lead denatures ALA dehydratase, which converts ALA to porphobilinogen.
Incorrect Responses
Option A (Decreased serum ferritin) is incorrect. Serum ferritin is elevated in lead poisoning.
Option B (Decreased zinc RBC protoporphyrin) is incorrect. Zinc RBC protoporphyrin levels are
also increased by lead poisoning and also in other conditions (e.g., iron deficiency).
Option C (Increased HbA2 and HbF) is incorrect. Lead does not interfere with globin chain
synthesis.
47.
A 32-year-old male complains of skin lesions that develop whenever his skin is exposed to
sunlight. The problem is so severe that he always avoids sunlight. He also notes that his urine has become
a wine-red color. Physical examination shows numerous fluid-filled vesicles on areas of skin most
exposed to sunlight. Laboratory studies, including a complete blood cell count and electrolytes, are
normal. He has a history of chronic hepatitis C. Which of the following is most likely responsible for the
color change in the urine?
A
Heme
B.
Urobilin
C.
Porphobilin
D.
Uroporphyrin I
Correct Response
Option D (uroporphyrin I) is correct. Uroporphyrin I gives urine a wine-red color, a characteristic
symptom of porphyria cutanea tarda. The disorder may be acquired by patients with hepatitis C.
Incorrect Responses
Option A (heme) is incorrect. Heme from hemoglobin or myoglobin is red before exposure to light. Heme
is also unlikely since hemoglobin levels are normal and muscle damage is not reported.
Option B (urobilin) is incorrect. Urobilin is the pigment normally responsible for the yellow color of
urine. An increase in urobilin causes the urine to be dark yellow. It does not turn wine-red color upon
exposure to light.
Option C (porphobilin) is incorrect. Porphobilin is increased in acute intermittent porphyria, an autosomal
recessive disease caused by a deficiency of uroporphyrinogen I synthase.
48.
A 4-year-old male has been deaf since birth and has a history of multiple fractures. Physical
examination reveals a bluish discoloration of the sclera. Which of the following proteins is most likely
defective?
A
fibrillin
B.
a-l-iduronidase
C.
type I collagen
D.
type III collagen
E.
Elastin
Correct Response
Option C Type I collagen defects produce osteogenesis imperfecta. Symptoms include deafness is due to
malformation of the auditory ossicles, bluish discoloration of the sclera, and bone fragility.
Incorrect Responses
Option A Fibrillin defects are associated with Marfan syndrome which does not have bone fractures or
blue sclera.
-L-iduronidase deficiency produces Hurler's syndrome which has skeletal deformities, but
there is no bone fragility or blue sclera.
Option D Type III collagen defects lead to Ehlers-Danlos syndrome characterized by vascular weakness.
Option E Elastin defects produce Williams syndrome which produces a supravalvular aortic stenosis.
49.
A 58-year-old female complains that her heart does “flip-flops” that keep her awake at night.
Physical examination reveals bilateral exophthalmos, enlarged thyroid, and pretibial myxedema. Which of
the following glycosaminoglycans is producing the pretibial myxedema?
A
Chondroitin sulfate
B.
Dermatan sulfate
C.
Heparin
D.
Hyaluronic acid
E.
Keratan sulfate I
Correct Response
Option B Pretibial myxedema results from excessive deposition of the glycosaminoglycan dermatan
sulfate in the interstitial tissue. This patient has Graves' disease which is characterized by myxedema.
Incorrect Responses
Option A Chondroitin sulfate is found principally in cartilage and is not found in interstitial tissue.
Option C Heparin is found in the granules of mast cells and in liver, lung, and skin, and is not found in
interstitial tissue.
Option D Hyaluronic acid is the major component of synovial fluid, where it serves as a joint lubricant
and is not found in interstitial tissue.
Option E Keratan sulfate I is found in cornea where it is associated with chondroitin sulfate and is not
found in interstitial tissue.
50.
A 28-year-old man has a family history of a disabling disorder that has caused the deaths of his
father, grandfather, and two paternal aunts. The father and aunts died at a younger age than did his
grandfather, and it appears that the condition progressively worsens with each generation. What disorder
is most likely?
A.
B.
C.
D.
E.
Cystic fibrosis
Duchenne muscular dystrophy
Huntington disease
Metachromatic leukodystrophy
Parkinson disease
Correct Response
Option C (Huntington disease) is correct. Huntington disease is an example of a triplet repeat
disorder with an amplification of the CAG trinucleotide in the translated region of the gene. More triplet
repeats occur with each generation, leading to earlier appearance of the disease (anticipation). The
disorder is autosomal dominant and affects both men and women.
Incorrect Responses
Option A (cystic fibrosis) is incorrect. Cystic fibrosis is caused by a defective protein known as
cystic fibrosis transmembrane conductance regulator. The condition has an autosomal recessive pattern of
inheritance; it is not a trinucleotide repeat disorder.
Option B (Duchenne muscular dystrophy) is incorrect. Duchenne muscular dystrophy is caused
by an absence or defect of the protein dystrophin, which is a membrane protein important in maintaining
the integrity of the muscle fiber. Duchenne muscular dystrophy has an X-linked pattern of inheritance; it
is not a trinucleotide repeat disorder.
Option D (metachromatic leukodystrophy) is incorrect. The primary cause of metachromatic
leukodystrophy is the absence of lysosomal arylsulfatase A activity, resulting in an accumulation of
sulfolipids in lysosomes. The condition has an autosomal recessive pattern of inheritance; it is not a
trinucleotide repeat disorder.
Option E (Parkinson disease) is incorrect. Parkinson disease is caused by loss of the neurons that
generate dopamine in the substantia nigra. Some forms of the disease are autosomal dominant, but none
has yet shown a tendency to occur prior to the expected onset.
51.
A cell culture of a tumor biopsy is suspected of containing a ras oncogene. Which protein is most
likely to confirm this diagnosis?
A.
B.
C.
D.
G protein
Growth factor
Growth factor receptor
Nuclear transcription factor
Correct Response
Option A (G protein) is correct. Ras is a mutated G protein that has lost its "off" switch, which is
a GTPase. The increase in signaling that results from a permanently activated G protein leads to
generation of a "growth" signal and unregulated growth of the cell line, which often leads to cancer.
Incorrect Responses
Option B (growth factor) is incorrect. An altered growth factor (e.g., platelet-derived growth
factor) is a product of a mutated gene for that growth factor, leading to overproduction (e.g., sis
oncogene), which may result in an astrocytoma or osteogenic sarcoma.
Option C (growth factor receptor) is incorrect. An altered growth factor receptor (e.g., epidermalderived growth factor, which is a mutated erb-B2 oncogene) produces a receptor that is active in the
absence of the growth factor. Altered growth factor may result in breast cancer.
Option D (nuclear transcription factor) is incorrect. A mutated nuclear transcription factor (e.g., a
mutated myc oncogene) may result in Burkitt lymphoma or neuroblastoma.
52.
A 48-year-old woman with type 2 diabetes mellitus complains of intermittent problems with
blurry vision. Physical examination shows a normal lens and mild diabetic retinopathy. What would best
evaluate the possibility that chronically elevated glucose levels contribute to this patient’s condition?
A.
B.
C.
D.
Blood glucose (fasting)
Blood ketones
Hemoglobin A1c concentration
Serum C-peptide
Correct Response
Option C (hemoglobin A1c concentration) is correct. Blurry vision in this patient most likely
results from poor glycemic control and the effect of glucose conversion to sorbitol by aldose reductase in
the lens. Sorbitol, which is osmotically active, causes water to move into the lens with subsequent
alteration in its refraction to light. Hemoglobin A1c measures the average level of glucose in the blood
over the previous 4 to 8 weeks; it is the product of a concentration-dependent nonenzymatically catalyzed
condensation of glucose and hemoglobin.
Incorrect Responses
Option A (blood glucose) is incorrect. Blood glucose only provides information about insulin
levels over the previous 4 to 6 hours.
Option B (blood ketones) is incorrect. Blood ketones are not an indicator of glycemic control.
They are an indication that insulin levels are very low or absent.
Option D (serum C-peptide) is incorrect. C-peptide levels are an indicator of endogenous insulin
53.
A 53-year-old farmer has a raised crateriform lesion on the inner aspect of the bridge of his nose
and numerous gray-white hyperkeratotic lesions on the face, hands, back of the neck, and forearms. What
is most likely responsible for the pathogenesis of these lesions?
A.
B.
C.
D.
Cleavage of DNA strands
Defects in DNA repair enzymes
Point mutations resulting in stop codons
Pyrimidine dimers resulting in distortion of the DNA helix
Correct Response
Option D (pyrimidine dimers resulting in distortion of the DNA helix) is correct. Ultraviolet B
(UVB) light (nonionizing radiation) is responsible for sunburn and the rapid increase of skin cancers in
the United States. UVB light excites adjacent pyrimidine bases on DNA strands, causing them to form
thymine-thymine dimers. In this patient, the crateriform lesion on the nose is a basal cell carcinoma, the
most common cancer caused by excessive exposure to UVB light. The gray-white hyperkeratotic lesions
on sun-exposed areas are actinic (solar) keratoses, which are precursors of squamous cell carcinoma of
the skin.
Incorrect Responses
Option A (cleavage of DNA strands) is incorrect. Ionizing radiation is responsible for cleavage of
DNA strands.
Option B (defects in DNA repair enzymes) is incorrect. Defects in DNA repair enzymes are more
often inherited than acquired. Xeroderma pigmentosum is an autosomal recessive disease characterized
by a deficiency of DNA repair enzymes.
Option C (point mutations resulting in stop codons) is incorrect. These nonsense mutations are
-thalassemia).
54.
A 3-year-old black girl is brought to a physician’s office for a routine physical examination.
During the examination, pallor of the conjunctiva and mucous membranes is observed. The patient is of
normal height for her age, but her weight is below normal. The girl’s cousin has sickle cell disease. A
PCR-based restriction (polymerase chain reaction-based) analysis produces three bands of 103 bp, 191bp,
-globin DNA, and a single
234-bp band is diagnostic for sickle cell anemia, what is the most likely diagnosis?
A.
B.
C.
D.
E.
Further studies are necessary
Results of blood studies are normal
Sickle cell disease
Sickle cell trait
-Thalassemia minor
Correct Response
Option D (sickle cell trait) is correct. Patients with sickle cell trait are most often asymptomatic and carry
-globin gene
sickle cell gene causes the replacement of gluta
-globin. Because of
this mutation, the PCR product cannot be cleaved by the restriction enzyme and a single band results at
294 bp. Because patients with sickle cell trait have one normal and one sickle cell gene, bands are seen at
294 bp, 103 bp, and 191 bp. The appearance of this single 294-bp band alone is diagnostic for sickle cell
anemia.
Incorrect Responses
Option A (further studies are necessary) is incorrect. Sickle cell trait can be diagnosed by PCR-based
restriction analysis for the sickle cell gene.
Option B (results of blood studies are normal) is incorrect. Healthy patients have bands at 103 bp and 191
bp only.
Option C (sickle cell disease) is incorrect. Sickle cell patients have a single 294-bp band.
Op
-thalassemia minor, the mutation occurs in the
-globin. The
electrophoresis reveals bands at 103 bp and 191 bp that are charact
-globin genecoding sequence. There is no single 294-bp band, which is diagnostic for sickle cell trait or sickle cell
anemia.
55.
A 50-year-old man complains of weight loss, fatigue, weakness, and a sense of fullness in the left
upper abdominal quadrant. Physical examination shows pallor, splenomegaly, and hepatomegaly; there is
no lymphadenopathy. Bone marrow aspirate shows 30% blasts. Laboratory studies show a leukocyte
count of 300,000/mm3 and the Philadelphia chromosome. What gene is most likely associated with these
findings?
A.
B.
C.
D.
E.
abl
brcal
c-myc
p53
Rb
Correct Response
Option A (abl) is correct. The patient’s clinical manifestations and laboratory findings are typical of
chronic myelogenous leukemia. This type of leukemia is characterized by overproduction of granulocytes,
especially neutrophils, leading to very high leukocyte counts and a marked splenomegaly. A characteristic
abnormality is the presence of the Philadelphia chromosome. This chromosome is a result of the
translocation of the proto-oncogene c-abl on chromosomes 9 to 22. The breakpoint on chromosome 22
occurs within the bcr (breakpoint cluster region) gene. The result is the chimeric gene bcr-abl, which is
expressed and encodes a novel tyrosine kinase.
Incorrect Responses
Option B (brca1) is incorrect. The brca1 gene is a tumor suppressor gene, which is missing in about 10%
of patients with breast cancer.
Option C (c-myc) is incorrect. The c-myc oncogene is characteristic of Burkitt lymphoma, a disease that
occurs mainly in children, with the mean age of onset of 7 years. In Burkitt lymphoma, the c-myc
oncogene from chromosome 8 is translocated into the heavy-chain-joining region of the immunoglobulin
gene (Jh) on chromosome 14. This results in the c-myc oncogene fusing with the Jh gene and being under
the control of the Jh promoter. The product of this fusion gene binds to DNA and stimulates further cell
division.
Option D (p53) is incorrect. The p53 gene is a tumor suppressor gene. Mutations in the p53 gene are the
most common DNA abnormality in tumors, occurring in more than 50% of cancers. Mutated p53
produces a defective 53-kd protein that is unable to induce apoptosis in a cell with damaged DNA.
Option E (Rb) is incorrect. The Rb gene is the retinoblastoma gene, a tumor suppressor gene. The absence
of Rb is associated with retinoblastoma, lung cancer, and other tumors. Homozygous individuals with two
defective copies of the Rb gene have a virtual certainty of developing a malignant retinoblastoma within
the first few months of life.
56.
A 48-year-old African-American woman who has widespread thickening of the skin is diagnosed
with diffuse cutaneous scleroderma. Laboratory studies show the presence of autoantibodies to a nuclear
enzyme that relieves torsional stress in DNA. This patient most likely produces autoantibodies to what
enzyme?
A.
Endonuclease
B.
C.
D.
E.
Exonuclease
Helicase
Ligase
Topoisomerase I
Correct Response
Option E (topoisomerase I) is correct. Scleroderma is a connective tissue disease characterized by fibrosis
of the skin and multiple internal organs, small-vessel vasculopathy, and specific autoimmune response
associated with autoantibodies. Up to 40% of patients with diffuse scleroderma develop antibodies to
topoisomerase I. Other autoantibodies seen in patients with scleroderma include those against centromere;
RNA polymerases I, II, and III; endoribonuclease; and U1snRNP. Topoisomerase I relieves torsional
stress in DNA by inducing reversible single-strand breaks in front of the replication fork.
Incorrect Responses
Option A (endonuclease) is incorrect. Endonucleases cleave nucleotides at internal positions in the
polynucleotide (DNA or RNA).
Option B (exonuclease) is incorrect. Exonucleases cleave mononucleotides one at a time from the ends of
a polynucleotide.
Option C (helicase) is incorrect. Helicase unwinds the DNA double helix in front of the replication fork,
causing positive supercoiling in front of the replication fork. This tight coiling must be removed by
topoisomerase for the replication fork to proceed.
Option D (ligase) is incorrect. Ligase is an enzyme that joins breaks in the DNA strand by forming
phosphodiester bonds.
57.
The parents of a 2-year-old child bring their son to the pediatrician’s office during a family
vacation in Florida. They have noticed that after playing on the beach for several days, the boy has
developed blisters and redness on his face, extremities, and trunk. He also has multiple freckles over the
face, lips, and extremities, as well as pigmented patches interspersed with hypopigmented areas. What
process is most likely to be defective in this child?
A.
B.
C.
D.
E.
DNA repair
DNA replication
mRNA splicing
Protein translation
RNA transcription
Correct Response
Option A (DNA repair) is correct. This child’s symptoms are characteristic of xeroderma pigmentosum. It
is a rare, heterogenous group of diseases in which patients have a hereditary deficiency of an enzyme that
repairs ultraviolet light–induced damage to DNA in keratinocytes and melanocytes. The most common
form of the disease is caused by the absence of the ultraviolet-specific endonuclease that normally
removes pyrimidine dimers. Extreme sensitivity to ultraviolet light, multiple freckles, and eventually skin
cancer result from the inability of these patients to maintain the integrity of DNA. Individuals with this
disorder should avoid exposure to sunlight.
Incorrect Responses
Option B, Option C, Option D, and Option E are incorrect. Refer to the discussion for Option A.
58.
A 20-year-old healthy woman with a family history of hereditary nonpolyposis colorectal cancer
(HNPCC) has genetic testing and is found to be a carrier of the HNPCC gene. Genetic testing shows a
mutation in a gene encoding a DNA repair protein. This protein most likely participates in which type of
DNA repair?
A.
B.
C.
D.
E.
Base excision
Depurination
Nucleotide excision
Postreplication mismatch
Recombinational
Correct Response
Option D (postreplication mismatch) is correct. HNPCC is caused by mutations in four genes that encode
proteins involved in DNA mismatch repair. DNA mismatch repair proteins are an excision system that
recognizes mismatched bases incorporated during DNA replication. Many such mismatched bases are
removed by the proofreading activity of DNA polymerase. Those that are missed are subject to later
correction by the DNA mismatch repair system. HNPCC is one of the most commonly inherited diseases,
affecting as many as 1 in 200 individuals. It accounts for up to 15% of all colon cancers.
Incorrect Responses
Option A (base excision) is incorrect. Base excision repair functions correctly in individuals with
HNPCC. This repair system involves several enzymes called DNA glycosylases. These enzymes remove
damaged pyrimidine or purine bases from the DNA, creating an apyridimic or apurinic site (an AP site) in
DNA. Remaining sugars are removed by AP endonucleases and phosphodiesterases. The gap of a single
nucleotide is then filled by DNA polymerase and ligase.
Option B (depurination) is incorrect. The depurination repair system is intact in individuals with HNPCC.
Depurination is a spontaneous hydrolytic chemical reaction known to create serious DNA damage in
cells. About 5,000 purine bases are lost per day from the DNA of each human cell. Depurination removes
a purine base (adenine and guanine), leaving a deoxyribose sugar. This sugar is removed by AP
endonucleases and phosphodiesterases, and the DNA sequence is restored by DNA polymerase and DNA
ligase.
Option C (nucleotide excision) is incorrect. This system removes bulky lesions, such as pyrimidine
dimers caused by sunlight or DNA bases ligated to large hydrocarbons. The nucleotide excision repair
system scans for DNA distortion rather than for a specific base change. An entire region of DNA
surrounding the lesion is removed by DNA helicase and then repaired by DNA polymerase and DNA
ligase. This system is defective in individuals with xeroderma pigmentosum.
Option E (recombinational) is incorrect. Recombinational repair depends on one strand of
parental DNA being undamaged. The gap in the DNA molecule synthesized on the damaged DNA
molecule can be filled with a DNA fragment from the undamaged parental DNA by recombination.
59.
A child of healthy parents has an autosomal recessive disorder. The 0.5 kb DNA fragment of the
normal gene amplified by polymerase chain reaction (PCR) contains a BamHI restriction site. On
digestion with BamHI, this fragment yields two fragments, which are 0.2 kb and 0.3 kb long. This
restriction site is lost in the mutated gene; thus PCR amplification yields a 0.5 kb DNA fragment that is
not sensitive to digestion with BamHI. The PCR-amplified 0.5 kb DNA fragments from both parents are
subjected to BamHI treatment and electrophoresis. Which of the following DNA patterns is most likely
seen on the gel?
A.
B.
DNA of at least one parent has no 0.5 fragment
DNA of at least one parent has neither 0.2 nor 0.3 kb fragments
C.
D.
E.
DNA of each parent separates into 0.2, 0.3, and 0.5 kb fragments
Maternal DNA migrates as a single 0.5 kb fragment
Paternal DNA migrates as a single 0.5 kb fragment
Correct Response
Option C (DNA of each parent separates into 0.2, 0.3, and 0.5 kb fragments) is correct. This child has an
autosomal recessive disorder, which is manifested when there is homozygosity for a mutant allele. Both
parents of the affected offspring are obligate heterozygotes and are healthy, with no features to show that
they are carriers. Each parent has one normal and one mutant allele. Thus, the DNA of each parent
separates on the electrophoretic gel into three fragments: 0.2, 0.3, and 0.5 kb.
Incorrect Responses
Option A (DNA of at least one parent has no 0.5 fragment), Option D (maternal DNA migrates as a single
0.5 kb fragment), and Option E (paternal DNA migrates a single 0.5 kb fragment) are incorrect. If the
DNA of one parent had no 0.5 kb fragment, which is characteristic in a recessive trait, the child would
have normal alleles. A child cannot be affected with a recessive autosomal disease if one parent is not a
carrier of this recessive trait.
Option B (DNA of at least one parent has neither 0.2 nor 0.3 kb fragments) is incorrect. If the DNA of
one parent had neither 0.2 nor 0.3 kb fragments, which are characteristic for a normal allele, both alleles
of the child would be mutated. Thus, the child would not be healthy.
60.
A 27-year-old man complains of genital sores. The patient reports recent unsafe sexual contact.
Examination of fluid from a bulla reveals multinucleated giant cells with intranuclear inclusions (Tzank
cells). Which of the following enzymes is the best target for therapy?
A
Adenosine deaminase
B.
Hypoxanthine-guanine phosphoribosyltransferase (HGPRT)
C.
Phosphoribosylpyrophosphate (PRPP) synthase
D.
Thymidine kinase
E.
Xanthine oxidase
Correct Response
Option D (thymidine kinase) is correct. This patient has a herpesvirus infection, most likely type 2. The
medication prescribed by the physician was probably acyclovir (or a more recent derivative). This agent
mimics a nucleoside and must be phosphorylated by thymidine kinase to block viral replication. Notably,
some virions of the herpes family have been found that do not contain thymidine kinase; therefore, they
are resistant to acyclovir and its congeners.
Incorrect Responses
Option A (adenosine deaminase) is incorrect. Adenosine deaminase is responsible for the conversion of
adenosine to inosine in the degradation pathway of adenosine. A deficiency of this enzyme leads to a
form of severe combined immunodeficiency disease (SCID).
Option B (hypoxanthine-guanine phosphoribosyltransferase) is incorrect. HGPRT is responsible for the
salvage of purines by converting hypoxanthine and guanine to their monophosphated forms. Lack of
HGPRT leads to destruction of free hypoxanthine and guanine, increased uric acid levels, ensuing mental
retardation, and, less importantly, gout.
Option C (phosphoribosylpyrophosphate synthase) is incorrect. PRPP synthase, which catalyzes the ratelimiting step in the synthesis of purine nucleotides, plays an important role in many other pathways. The
most common alteration of PRPP synthase is overactivity, which leads to an overproduction of purines
and the development of hyperuricemia and gout.
Option E (xanthine oxidase) is incorrect. Xanthine oxidase, a terminal enzyme in the degradation pathway
of purines, helps convert xanthine to uric acid. Allopurinol, which decreases the production of uric acid as
well as the frequency of gout in individuals prone to attacks, may inhibit this enzyme.
61.
A 59-year-old male complains of a progressively worsening cough, drenching night sweats, and
increasing fatigue. He is diagnosed with Mycobacterium tuberculosis infection. The patient is prescribed
a four-drug regimen, including rifampin. Which of the following bacterial sites is inhibited by the
rifampin?
A
acyl carrier protein
B.
RNA polymerase
C.
arabinosyl transferase
D.
ribosomal protein
Correct Response
Option B Rifampin is a first-line drug in the treatment of tuberculosis. Its mode of action involves
inhibition of RNA polymerase.
Incorrect Responses
Option A Isoniazid blocks an acyl carrier protein in mycobacteria, but not rifampin
Option C Ethambutol inhibits arabinosyl transferase in mycobacteria, but not rifampin.
Option D Streptomycin inhibits ribosomal assembly by binding to a 30S subunit protein in mycobacteria,
but not rifampin.
62.
A three year old male is brought to the pediatrician because he has begun to stumble and has
obvious difficulty running, jumping, and climbing stairs. His calf muscles have also become noticeably
enlarged. Deletion scanning PCR revealed an absent band from the dystrophin gene after electrophoresis
and blotting. What does the missing band represent?
A
promoter
B.
exon
C.
enhancer
D.
splice site
Correct Response
Option B The mutation that causes Duchenne muscular dystrophy is a deletion in one of the exons that
produces a dysfunctional dystrophin protein.
Incorrect Responses
Option A The mutation that causes Duchenne muscular dystrophy is a deletion in one of the exons, not in
the promoter site.
Option C The mutation that causes Duchenne muscular dystrophy is a deletion in one of the exons, not in
an enhancer.
Option D The mutation that causes Duchenne muscular dystrophy is a deletion in one of the exons, not in
a splice site.
63.
A 24-year-old man returns from a trip to India and has profuse, watery diarrhea and a feeling of
lightheadedness when he stands. A Gram stain of stool shows comma-shaped gram-negative organisms
and an absence of an inflammatory reaction. What most accurately describes the biochemical alterations
underlying this patient’s diarrhea?
A.
B.
C.
D.
Activation of a Gq protein
Activation of a Gs protein
Inactivation of a Gi protein
Inhibition of a Cl¯ channel
E.
Stimulation of guanylyl cyclase activity
Correct Response
Option B (activation of a Gs protein) is correct. The history and stool findings in this patient are
characteristic of cholera caused by Vibrio cholerae. The cholera toxin acts on the intestinal mucosa and
produces diarrhea by covalently modifying a Gs protein by ADP ribosylation. This is effective in
permanently turning the protein "on," thus elevating intracellular levels of cAMP. This action stimulates
the opening of chloride channels, resulting in a secretory diarrhea. Cholera is a toxin-induced disease and
has no inflammatory component. Treatment involves fluid and electrolyte replacement to prevent
dehydration (manifested as orthostatic symptoms in this case) and electrolyte imbalance.
Incorrect Responses
Option A (activation of a Gq protein) is incorrect. Gq proteins activate phospholipase C, which
cleaves phosphatidylinositol diphosphate to diacylglycerol and inositol triphosphate, in turn raising
intracellular calcium concentrations. Examples of Gq receptors are type 1 muscarinic cholinergic
-adrenergic receptors in the autonomic nervous system.
Option C (inactivation of a Gi protein) is incorrect. Gi proteins, when activated, inhibit the
formation of cAMP. Permanent inactivation of the Gi protein, which leads to elevated levels of cAMP, is
the mechanism of action of the pertussis toxin in respiratory epithelium.
Option D (inhibition of a Cl channel) is incorrect. The Cl channel, the final step in the generation
of secretions from part of the intestines, is activated in this patient. Inhibition of the channel leads to a
lack of secretions; this condition is often seen in patients with defective channels (e.g., with cystic
fibrosis).
Option E (stimulation of guanylyl cyclase activity) is incorrect. Guanylyl cyclase, which produces
cGMP from GTP, is part of the mechanism of action of a number of molecules, including the atrial
natriuretic peptides and all agents that produce nitric oxide for vasodilation. Drugs, such as sildenafil,
inhibit the breakdown of cGMP, thereby increasing cGMP concentrations and producing vasodilatory
effects.
64.
After exercising strenuously for 1 hour, a 23-year-old healthy man drinks two glasses of wine. He
suddenly feels dizzy and has a rapid heartbeat and clammy skin. Which mechanism is most likely
responsible for this man’s reaction to alcohol?
A.
B.
C.
D.
E.
Gluconeogenesis is inhibited by the abundance of oxaloacetate
Gluconeogenesis is inhibited by the abundance of pyruvate
Glycogenolysis is inhibited by ethanol
NADH is used in ethanol metabolism
Oxaloacetate and pyruvate are diverted from the gluconeogenetic pathway
Correct Response
Option E (oxaloacetate and pyruvate are diverted from the gluconeogenetic pathway) is correct. Alcohol
is metabolized in the liver by two oxidation reactions that generate large quantities of NADH. Because
the reduction of pyruvate to lactate and oxaloacetate to malate are favored by the abundance of NADH,
pyruvate and oxaloacetate are diverted from gluconeogenesis. Individuals with depleted liver glycogen
stores, which occur during extended periods of strenuous exercise, are dependent on gluconeogenesis to
maintain adequate blood glucose levels. In these individuals, the consumption of ethanol can rapidly
result in hypoglycemia.
Incorrect Responses
Option A (gluconeogenesis is inhibited by the abundance of oxaloacetate) is incorrect. Oxaloacetate is an
intermediate in the synthesis of glucose by gluconeogenesis; thus its abundance does not inhibit
gluconeogenesis.
Option B (gluconeogenesis is inhibited by the abundance of pyruvate) is incorrect. Pyruvate is a precursor
of glucose produced in gluconeogenesis; thus abundance of pyruvate does not inhibit gluconeogenesis.
Option C (glycogenolysis is inhibited by ethanol) is incorrect. Ethanol metabolism has no effect on
glycogenolysis.
Option D (NADH is used in ethanol metabolism) is incorrect. NADH is not used but is generated in
alcohol metabolism. See discussion for Option E.
65.
An 18-year-old male complains of a rash and hair loss to his physician. He reports that in addition
to a balanced diet he eats about 20 raw eggs per day to enhance his body-building exercises. What
additional symptom is most likely in this patient?
A.
B.
C.
D.
E.
bleeding gums
dementia
glossitis
lactic acidosis
nystagmus
Correct Response
Option D (lactic acidosis) is correct. Raw eggs contain avidin, a protein that binds to biotin preventing its
absorption. Impaired pyruvate carboxylase activity leads to lactic acidosis.
Incorrect Responses
Option A (bleeding gums) is incorrect. Vitamin C deficiency produces bleeding gums, but not dermatitis
and hair loss seen in biotin deficiency.
Option B (dementia) is incorrect. Niacin deficiency produces dermatitis as part of the pellagra syndrome,
but not hair loss seen in biotin deficiency.
Option C (glossitis) is incorrect. Riboflavin deficiency produces glossitis, but not dermatitis and hair loss
seen in biotin deficiency.
Option E (nystagmus) is incorrect. Thiamine deficiency produces Wernicke-Korsakoff syndrome with
symptoms like confusion, ataxia, nystagmus, but not dermatitis and hair loss seen in biotin deficiency.
66.
A 62-year-old woman whose husband died recently has developed a vitamin deficiency due to
adopting a diet consisting of cereal and diet cola. She complains of bleeding gums after brushing her teeth
and pain in her legs when she walks. Physical examination shows a smooth red tongue, gingivitis, carious
teeth, scattered ecchymoses over the trunk, and pinpoint areas of hemorrhage around the hair follicles.
Which one of the following reactions most likely accounts for these physical findings?
A.
B.
C.
D.
E.
Cleavage of N- and C-terminal propeptide fragments
Formation of pro-α-chains
Glycosylation of side-chain residues
Hydroxylation of proline and lysine side-chains
Triple helix assembly of procollagen
Correct Response
Option D (hydroxylation of proline and lysine side-chains) is correct. Vitamin C, as well as
molecular oxygen and α-ketoglutarate, are the requirements for the proper function of prolyl hydroxylase,
the enzyme responsible for hydroxylation of the proline side-chains in collagen. Collagen lacking such
side-chain hydroxyl groups cannot be stabilized by interchain hydroxyl groups. This lowers the melting
point of collagen and weakens the connective tissues that contain it, leading to hemorrhage.
Incorrect Responses
Option A (cleavage of N- and C-terminal propeptide fragments) is incorrect. This process occurs
extracellularly, yielding the collagen molecule (monomer). The monomers later associate and are further
cross-linked by lysyl oxidase for stability while in the extracellular matrix. The cleavage process occurs
via propeptidases and does not depend on vitamin C.
Option B (formation of pro-α-chains) is incorrect. This step, which involves translation of the
mRNA to form the peptide chains found in the endoplasmic reticulum, does not depend on vitamin C.
Option C (glycosylation of side-chain residues) is incorrect. This step, which involves the
addition of glucose and galactose sugars to selected proline and lysine residues, does not depend on
vitamin C.
Option E (triple helix assembly of procollagen) is incorrect. This spontaneous process, which
occurs in the Golgi apparatus, yields a procollagen molecule and does not require vitamin C.
67.
A 42-year-old woman with severe rheumatoid arthritis begins taking an immunosuppressive drug.
A few weeks later, she develops moderately severe macrocytic anemia, with pancytopenia and
hypersegmented neutrophils. The most likely mechanism underlying this drug-induced anemia involves
inhibition of
A.
B.
C.
D.
Dihydrofolate reductase
Intestinal conjugase
Production of intrinsic factor
Reabsorption of folate
Correct Response
Option A (dihydrofolate reductase) is correct. The patient is most likely taking methotrexate,
which is a competitive inhibitor of dihydrofolate reductase. The lack of dihydrofolate reductase leads to a
lack of tetrahydrofolate, and therefore inhibits the conversion of dUMP to TMP. This process stops
proper DNA synthesis and occurs primarily in rapidly dividing cells, which are cells of hematopoietic
origin.
Incorrect Responses
Option B (intestinal conjugase) is incorrect. Intestinal conjugase is responsible for the conversion
of the polyglutamate form of folate to the more readily absorbed monoglutamate form. Inhibition of
intestinal conjugase is not a known side effect of immunosuppressive agents.
Option C (production of intrinsic factor) is incorrect. Intrinsic factor is secreted by the parietal
cells of the stomach and binds to vitamin B12 in foodstuffs. Intrinsic factor is required for proper
absorption of vitamin B12 in the terminal ileum. Lack of vitamin B12 can lead to a macrocytic anemia,
but it would be unlikely to do so within a few weeks. Furthermore, methotrexate does not interfere with
the production of intrinsic factor.
Option D (reabsorption of folate) is incorrect. Alcohol and oral contraceptives are known to
interfere with absorption of the monoglutamate form of folate in the jejunum.
68.
A 38-year-old woman with chronic alcoholism has been homeless for the past year. Her diet
consists primarily of starchy processed "junk" foods, in addition to alcoholic beverages. Physical
examination shows noticeably thickened skin and the presence of hyperkeratotic hair follicles. The
conjunctivae are dry, and tear production is reduced. What other finding is most likely to be expected?
A.
B.
C.
D.
E.
Bleeding gums
Hemolytic anemia
Impaired night vision
Osteomalacia
Pain in the long bones
Correct Response
Option C (impaired night vision) is correct. This patient shows signs of vitamin A deficiency
(thick skin, follicular hyperkeratosis, dry eyes), which are probably caused by a diet lacking in sources of
vitamin A (retinol), such as vegetables. Other than keratotic thickening of the epithelium, a major
symptom of this vitamin deficiency is impaired night vision as a result of the reduced synthesis of visual
purple.
Incorrect Responses
Option A (bleeding gums) is incorrect. A deficiency of vitamin C leads to defective collagen
formation and causes bleeding gums. This condition is often manifested by weakened mucous
membranes. Vitamin C (ascorbic acid) is found in citrus fruits, broccoli, tomatoes, and strawberries.
Option B (hemolytic anemia) is incorrect. Vitamin E deficiency causes hemolytic anemia.
Vitamin E prevents lipid peroxidation of cell membranes; its deficiency damages red blood cell
membranes. Vitamin E (tocopherol) is found in vegetable oils and wheat germ.
Option D (osteomalacia) is incorrect. Vitamin D deficiency causes osteomalacia, or softening of
the bones, a condition that leads to decreased mineralization of bone. Vitamin D (calcitriol) is found in
fortified milk or is manufactured in the body from 7-dehydrocholesterol.
Option E (pain in the long bones) is incorrect. Vitamin A overdose or vitamin D deficiency may
cause pain in the long bones, with associated osteomalacia and an increase in pathologic fractures.
Sources of vitamin A (retinol) are liver and yellow and dark green leafy vegetables.
69.
A 25-year-old medical student who has had diabetes mellitus since childhood is unconscious
when admitted to the emergency room. His wife states that he has been studying for an examination for
the past 2 days and has not been injecting his insulin on a regular basis. Physical examination shows a
blood pressure of 80/40 mm Hg, heart rate of 124 beats/min, and respiratory rate of 35 breaths/min. His
breath has a fruity odor, and his skin turgor is poor. What additional finding is most likely to be
increased?
A.
blood sodium
B.
blood lactate
C.
blood pH
D.
serum bicarbonate
E.
anion gap
Correct Response
Option E (anion gap) is correct. The patient is in a coma and has diabetic ketoacidosis. The
increased concentration of ketones in the blood will produce an increase in the calculated anion gap.
Incorrect Responses
Option A (blood sodium) is incorrect. The patient is in diabetic ketoacidosis, which is
characterized by sodium depletion.
Option B (blood lactate) is incorrect. The patient is in diabetic ketoacidosis, which is
-hydroxybutyrate, not lactate.
Option C (blood pH) is incorrect. The patient is in diabetic ketoacidosis, which is characterized by a
decrease in blood pH.
Option D (serum bicarbonate) is incorrect. The patient is in diabetic ketoacidosis, which is
characterized by bicarbonate depletion as it is converted to carbon dioxide to buffer the acidity.
70.
A 25-year-old man who has been homeless for several months is admitted to the hospital in a
state of starvation. He has not eaten anything in the past week. He is started on a diet in which 60% of the
calories come from carbohydrates. This diet would most likely increase the activity of what enzyme?
A.
B.
C.
D.
E.
Carnitine acyltransferase
Citrate synthase
Glucokinase
Glucose-6-phosphatase
HMG-CoA synthase
Correct Response
Option C (glucokinase) is correct. The restoration of ample carbohydrate to the diet would
increase insulin concentrations. Glucokinase, located in liver and pancreatic cells, is increased in
concentration and activity by insulin. Hexokinase, which is located in extrahepatic cells, is not regulated
by insulin.
Incorrect Responses
Option A (carnitine acyltransferase) is incorrect. Carnitine acyltransferase is already maximally
increased in order to oxidize fatty acids released during starvation.
Option B (citrate synthase) is incorrect. Citrate synthase is already maximally increased in order
to metabolize the acetyl CoA from fatty acid oxidation.
Option D (glucose-6-phosphatase) is incorrect. Glucose-6-phosphatase is already maximally
increased in order to release glucose produced in the liver via gluconeogenesis into the bloodstream.
Option E (HMG-CoA synthase) is incorrect. To adapt to starvation, HMG-CoA synthase, the
rate-limiting enzyme in ketogenesis, is already maximally increased in order to form ketone bodies.
71.
A 23-year-old woman, who has had diabetes mellitus since the age of 10, is unconscious when
admitted to the emergency room. Physical examination shows a blood pressure of 80/40 mm Hg, heart
rate of 124 beats/min, and respiratory rate of 35 breaths/min. She has poor skin turgor and a fruity odor to
her breath. Her blood glucose level is 750 mg/dL. What enzyme is most likely responsible for the
presence of a fruity odor on this patient’s breath?
A.
B.
C.
Acetyl-CoA carboxylase
Citrate lyase
HMG-CoA reductase
D.
E.
HMG-CoA synthase
Pyruvate dehydrogenase
Correct Response
Option D (HMG-CoA synthase) is correct. HMG-CoA synthase, the rate-limiting step in the
production of ketones from acetyl CoA, accounts for the fruity odor. The smell of acetone on the breath,
the signs of volume depletion (poor skin turgor), and the hyperglycemia indicate that the patient is in a
state of diabetic ketoacidosis. Lack of insulin leads to elevated levels of free fatty acids and increased
fatty acid oxidation in the liver, with concomitant ketogenesis.
Incorrect Responses
Option A (acetyl-CoA carboxylase) is incorrect. Acetyl-CoA carboxylase, the enzyme that forms
malonyl CoA, is the rate-limiting enzyme in the biosynthesis of fatty acids. Fatty acid oxidation is
occurring, and fatty acid synthesis is inhibited.
Option B (citrate lyase) is incorrect. Citrate lyase converts citrate to oxaloacetate and acetyl CoA,
generating cytosolic acetyl CoA for the synthesis of fatty acids. This system is inactive during times of
fatty acid oxidation.
Option C (HMG-CoA reductase) is incorrect. HMG-CoA reductase is related to cholesterol
metabolism and not to ketone metabolism. It converts HMG-CoA to mevalonate.
Option E (pyruvate dehydrogenase) is incorrect. This patient’s symptoms are the result of
excessive ketone production, which decreases the activity of this enzyme. Acetyl CoA inhibits pyruvate
dehydrogenase.
72.
During a physical examination of a 24-year-old man who is mentally retarded, the physician
notices clumsiness and spasticity of the patient's limbs, emotional lability, and decreased visual-spatial
discrimination. Laboratory studies show spherical granular masses in urinary sediment that stain positive
with PAS and Alcian blue. The patient most likely has a deficiency of what enzyme?
A.
B.
C.
D.
E.
Arylsulfatase A
-Glucosidase
Hexosaminidase A
Muscle phosphorylase
Sphingomyelinase
Correct Response
Option A (arylsulfatase A) is correct. A deficiency of arylsulfatase results in metachromatic
leukodystrophy, an autosomal recessive disease. The disorder causes an accumulation of a sulfatecontaining ceramide (positive PAS and Alcian blue stain) in lysosomes. Demyelination of nerve cells
leads to mental retardation and motor abnormalities, which are the most common symptoms of
arylsulfatase deficiency.
Incorrect Responses
-glucosidase results in Gaucher disease,
an autosomal recessive disease. This disorder is the most frequently occurring form of sphingolipidosis. It
causes accumulation of ceramide with an attached glucose moiety and has different staining properties
and tissue localization than metachromatic leukodystrophy. Gaucher cells in the adult form are found in
the reticuloendothelial system. Presenting symptoms may include mental retardation, but there are no
signs of adverse effects on myelin production.
Option C (hexosaminidase A) is incorrect. A deficiency of hexosaminidase A results in TaySachs disease, an autosomal recessive disorder, causing the accumulation of GM2 ganglioside. Symptoms
of Tay-Sachs disease include mental retardation and blindness.
Option D (muscle phosphorylase) is incorrect. A deficiency in muscle phosphorylase results in
McArdle disease, an autosomal recessive disorder. The disease causes the loss of an immediate source of
glucose to support muscle contraction, which leads to muscle cramping with exercise (a common
presenting symptom).
Option E (sphingomyelinase) is incorrect. A deficiency in sphingomyelinase results in NiemannPick disease, an autosomal recessive disorder. This disorder causes sphingomyelin to accumulate in the
lysosomes, which produces symptoms such as liver and spleen enlargement in addition to mental
retardation. It is usually fatal early in life.
73.
An otherwise healthy 1-year-old girl has fasting hypoglycemia after eating a meal containing
carbohydrates. Extended fasting does not result in ketosis or lactic acidosis. The patient most likely has
what disorder?
A.
B.
C.
D.
Glycogen storage disease type VI
Medium-chain acyl CoA dehydrogenase deficiency
Pyruvate carboxylase deficiency
Thiamine deficiency
Correct Response
Option B (medium-chain acyl CoA dehydrogenase deficiency) is correct. Medium-chain acyl
CoA dehydrogenase deficiency leads to abnormal formation of carnitine esters excreted in the urine. The
resulting carnitine deficiency prevents both the utilization of long-oxidation for fuel
-oxidation of fatty
acids. The reduced availability of fatty acids for oxidation leads to a reduction in gluconeogenesis,
resulting in hypoglycemia.
Incorrect Responses
Option A (glycogen storage disease type VI) is incorrect. Glycogen storage disease type VI (Hers
disease) is caused by a deficiency in liver glycogen phosphorylase, which also produces a fasting
hypoglycemia. However, the activation of the gluconeogenic pathway and mobilization of fatty acids to
provide energy leads to ketosis in these patients.
Option C (pyruvate carboxylase deficiency) is incorrect. Pyruvate carboxylase is a gluconeogenic
enzyme that helps convert pyruvate to oxaloacetate. A deficiency of this enzyme leads to lactic acidosis,
because pyruvate is unable to be otherwise used in the gluconeogenic pathway.
Option D (thiamine deficiency) is incorrect. Thiamine deficiency causes defective operation of
pyruvate dehydrogenase, leading to an accumulation of pyruvate and subsequent lactic acidosis.
74.
A 65-year-old man with a known history of chronic alcoholism complains of severe chest pain
that radiates down the inside of his left arm. An ECG shows a prior acute anterior myocardial infarction
(MI). A complete blood cell count shows a macrocytic anemia with hypersegmented neutrophils and mild
pancytopenia. No neurologic abnormalities are evident. The plasma homocysteine level is increased.
What best explains the increase in plasma homocysteine in this patient?
A.
Decreased level of cystathionine synthase
B.
C.
D.
E.
Decreased level of dihydrofolate reductase
Decreased level of thymidylate synthetase
Decreased serum folate
Decreased serum vitamin B12
Correct Response
Option D (decreased serum folate) is correct. Alcoholism is the most common cause of folate
deficiency. Alcoholism accompanied by a poor diet leads to decreased liver stores of folate and decreased
reabsorption of the monoglutamate form of folate in the jejunum. Vitamin B12 (cobalamin) removes the
methyl group from N5-methyltetrahydrofolate (circulating form of folate) to produce methyl–vitamin B12
and tetrahydrofolate. The methyl group is then transferred to homocysteine to produce methionine.
Therefore, a decrease in either N5-methyltetrahydrofolate or vitamin B12 results in increased plasma
homocysteine levels. An increase in plasma homocysteine levels leads to damage of endothelial cells and
the potential for vessel thrombosis, which may have played a role in this patient’s MI.
Incorrect Responses
Option A (decreased level of cystathionine synthase) is incorrect. Deficiency of cystathionine
synthase is present in homocystinuria, a rare autosomal recessive disease. Cystathionine synthase
catalyzes the reaction that converts homocysteine to cystathionine. Elevated plasma homocysteine and
methionine (regenerated from homocysteine) as well as a propensity for vessel thrombosis are
characteristic of this disorder.
Option B (decreased level of dihydrofolate reductase) is incorrect. Dihydrofolate reductase
converts oxidized dihydrofolate back to tetrahydrofolate. The amount of dihydrofolate reductase increases
during deficiency of folate by gene amplification.
Option C (decreased level of thymidylate synthetase) is incorrect. Thymidylate synthetase
converts deoxyribose uridine monophosphate to deoxyribose thymidine monophosphate, which is used in
DNA synthesis. This enzyme requires folate, and although the amount of enzyme present is not
decreased, its activity will be decreased.
Option E (decreased serum vitamin B12) is incorrect. Vitamin B12 deficiency produces
neurologic abnormalities in the spinal cord and central nervous system. Because there are no neurologic
findings in this case, vitamin B12 deficiency is unlikely.
75.
A 52-year-old man with chronic bronchitis requires continuous supplementary O2 to maintain an
adequate arterial PO2. What would most likely improve O2 delivery to this patient’s tissues?
A.
B.
C.
D.
Decreased arterial pH
Decreased red blood cell 2,3-bisphosphoglycerate (2,3-BPG)
Decreased temperature
Hyperventilation
Correct Response
Option A (decreased arterial pH) is correct. Hemoglobin exists in two forms: the relaxed, or R
form, which has high O2 affinity; and the taut, or T form, which has low O2 affinity. By stabilizing the T
form, acidosis decreases the affinity of hemoglobin for O2 (i.e., causes a right-shift of the O2-binding
curve), thus releasing more O2. This action is referred to as the Bohr effect.
Incorrect Responses
Option B (decreased red blood cell 2,3-bisphosphoglycerate) is incorrect. The T form of
hemoglobin is stabilized by 2,3-BPG, encouraging hemoglobin to release its O2 load. Decreased red
blood cell 2,3-BPG increases the affinity of hemoglobin for O2, causing a left shift of the O2-binding
curve.
Option C (decreased temperature) is incorrect. Elevated temperatures stabilize the T form of
hemoglobin. Therefore, decreased temperatures (hypothermia) increase the affinity of hemoglobin for O2,
causing the O2-binding curve to shift to the left.
Option D (hyperventilation) is incorrect. Hyperventilation increases the loss of CO2, causing
respiratory alkalosis and a left-shift of the O2-binding curve.
76.
A 3-year-old child shows signs of mental deterioration and skeletal deformities. Microscopic and
laboratory analyses show enlarged lysosomes and increased levels of lysosomal enzymes in the serum.
These findings are most likely caused by a defect in what process?
A.
B.
C.
D.
Gene amplification
Mannose 6-phosphate attachment
Nuclear localization signal
Transferrin receptor mRNA
Correct Response
Option B (mannose 6-phosphate attachment) is correct. This patient has I-cell (inclusion cell)
disease (also known as mucolipidosis II), which is caused by defective shuttling of lysosomal enzymes
into lysosomes. Enzymes destined for lysosomes normally receive a mannose 6-phosphate marker in the
Golgi apparatus. In I-cell disease, the enzymatic ability to phosphorylate mannose in the Golgi apparatus
is lacking, and enzymes are targeted at extracellular sites other than lysosomes. Because the lysosomes
lack enzymes, undegraded molecules accumulate in the lysosomes and form inclusion bodies.
Incorrect Responses
Option A (gene amplification) is incorrect. Gene amplification is the process of simultaneously
activating multiple copies of DNA fragments that code for the same RNA, thus leading to a drastic
increase in the concentration of the final product. This process occurs during both the development of
resistance to methotrexate (via amplification of the dihydrofolate reductase gene) and the early stages of
development in the ovum (primarily for ribosomal RNA needs). It does not play a role in the targeting of
lysosomal enzymes.
Option C (nuclear localization signal) is incorrect. The nuclear localization signal, which is a
sequence that contains five positively charged amino acids (usually lysine residues), directs proteins into
the nucleus. It does not influence the targeting of lysosomal enzymes.
Option D (transferrin receptor mRNA) is incorrect. Transferrin, a protein, is responsible for the
shuttling of iron in the body. The level of iron in cells directly influences the transcription of the
transferrin receptor gene. Therefore, low levels of iron lead to an increased expression of the receptor and,
hence, an increased uptake of iron via transferrin endocytosis. However, transferrin receptor mRNA is not
related to the targeting of lysosomal enzymes.
77.
A 25-year-old woman complains of chest pain. On physical examination, a midsystolic click is
audible, followed by a late systolic murmur, indicating a valvular disease. What glycosaminoglycan is
associated with the valvular lesion?
A.
B.
C.
D.
Chondroitin sulfate
Dermatan sulfate
Heparan sulfate
Hyaluronic acid
E.
Keratan sulfate
Correct Response
Option B (dermatan sulfate) is correct. This patient has mitral valve prolapse, which is
characterized by redundancy of the mitral valve leaflets, causing them to prolapse into the left atrium
during systole. This results in a click during prolapse, followed by a short systolic murmur because of
mitral regurgitation. The glycosaminoglycan dermatan sulfate is found in valvular tissue in the heart.
Glycosaminoglycans, the major components of ground substance in the interstitial tissue, are complexes
of branched, negatively charged polysaccharide chains that contain amino sugars (e.g., glucosamine or
galactosamine) and acid sugars (iduronic acid or glucuronic acid). Sections through the mitral valve in a
patient with mitral valve prolapse would show myxomatous degeneration involving loss of dermatan
sulfate.
Incorrect Responses
Option A (chondroitin sulfate) is incorrect. This glycosaminoglycan is found primarily in
cartilage.
Option C (heparan sulfate) is incorrect. This glycosaminoglycan is found primarily in the
glomerular basement membrane, where it is responsible for the strong negative charge that repels
albumin.
Option D (hyaluronic acid) is incorrect. This glycosaminoglycan is the major component of
synovial fluid, where it serves as a joint lubricant.
Option E (keratan sulfate) is incorrect. This glycosaminoglycan is found in cartilage, where it is
associated with chondroitin sulfate.
78.
A 40-year-old man of Scandinavian descent sees his physician with complaints of shortness of
breath and a nonproductive cough. He has smoked one pack of cigarettes daily for 30 years. A chest
radiograph shows increased lung volume, and the diaphragm appears flatter than normal. The suspected
diagnosis is emphysema because of an enzyme deficiency. This deficiency most likely results in
A.
B.
C.
D.
E.
decreased collagen cross-linking
defective collagen fiber synthesis
de
-keratin synthesis
increased neutrophil elastase
mutated elastin protein
Correct Response
Option D (increased neutrophil elastase) is correct. Toxic substances in cigarette smoke trigger a release
of inflammatory agents, attracting activated neutrophils to the alveolar region. Neutrophil elastase is
released by the activated neutrophils as a part of their antimicrobial action. In the absence of the inhibiting
-antitrypsin, neutrophil elastase remains active and destroys alveolar walls, causing
panlobular emphysema.
Incorrect Responses
Option A (decreased collagen cross-linking) is incorrect. Although the crosslinking of collagen is
-antitrypsin has no role in this process.
Option B (de
-Antitrypsin has no role in collagen fiber
synthesis.
-Keratin is found in hair, nails, and the outer
-Antitrypsin has no role in its synthesis.
-antitrypsin deficiency, neutrophil elastase cannot be
-antitrypsin does not cause a mutation
in elastin protein.
79.
A 3-year-old male with nephrotic syndrome has a urine protein level >4 g/day. Physical
examination reveals generalized edema and ascites. What best describes the effect on calcium in his
serum?
A.
B.
C.
D.
Decreased ionized calcium
Decreased total calcium
Normal total calcium
Increased ionized calcium
Correct Response
Option B (decreased total calcium) is correct. In nephrotic syndrome the loss of albumin in the urine
leads to hypoalbuminemia. Total serum calcium includes calcium bound to albumin (40%) and ionized
calcium.
Incorrect Responses
Option A (decreased ionized calcium) is incorrect. In nephrotic syndrome the ionized calcium fraction
remains at normal concentrations. The total calcium is reduced because of the removal of albumin
calcium binding capacity.
Option C (normal total calcium) is incorrect. Nephrotic syndrome creates a hypoalbuminemia that lowers
the total serum calcium by reducing the binding sites normally available on albumin.
Option D (increased ionized calcium) is incorrect. In nephrotic syndrome the ionized calcium fraction
remains at normal concentrations. The total calcium is reduced because of the removal of albumin
calcium binding capacity.
80.
A 23-year-old male is admitted to the emergency room after an evening of dancing at a rock
concert. He reports that he took a large number of “poppers” to increase his enjoyment at the concert. He
has a bluish discoloration to his lips, palms of his hands, and his fingernails. Arterial blood gas analysis
shows a normal p02 but the blood is a chocolate-brown color. Administration of pure O2 does not
improve the bluish discoloration. Which of the following is the most likely diagnosis?
A
glucose 6-phosphate deficiency
B.
carbon monoxide poisoning
C.
methemoglobinemia
D.
iron deficiency anemia
Correct Response
Option C Methemoglobinemia, caused by the amyl nitrate (poppers), has heme iron in the ferric state
which cannot bind O2 leading to cyanosis and chocolate-brown color of the blood.
Incorrect Responses
Option A Several drugs such as salicylates can precipitate a hemolytic anemia in glucose 6-phosphate
deficiency but not amyl nitrite.
Option B carbon monoxide poisoning produces a cherry red appearance in some severe cases, and it is
characterized by headache, dizziness, and nausea, none of which were evident in this patient.
Option D Iron deficiency would produce a pallor to the skin, not cyanosis.
81.
A 15 year old female complains of fatigue. Her laboratory results indicate her serum iron is 7.5
mmole/liter. Which of the following will most likely be elevated by the indicated therapy?
A
transferrin
B.
ceruloplasmin
C.
ferritin
D.
haptoglobin
E.
albumin
Correct Response
Option A This patient has iron deficiency anemia and requires iron supplements. Transferrin is a serum
protein that transports iron to the tissues.
Incorrect Responses
Option B Ceruloplasmin oxidized iron in the serum to Fe+++ which is the form that binds to transferrin.
Option C Ferritin is the protein apoferritin with iron bound to it. It is a storage protein found in the
tissues.
Option D Haptoglobin is a plasma glycoprotein that binds free (extracorpuscular) hemoglobin preventing
it from passing through the glomerulus.
Option E Albumin is not involved in the transport of iron.
82.
A 45 year old woman developed dementia, myoclonus, weakness and spasticity. She became
comatose within months after onset of neurological signs. Postmortem brain biopsy showed spongiform
encephalopathy consistent with Creutzfeldt-Jakob disease. Several encephalopathies including
Creutzfeldt-Jakob are thought to be caused by prions. What level of protein structure is altered by prions?
A
primary structure
B.
secondary structure
C.
tertiary structure
D.
quaternary structure
Correct Response
Option B Prions induc
-structure in the native protein in neuronal membranes.
This creates a secondary alteration in tertiary structure.
Incorrect Responses
-globin. Prions do
not affect primary structure.
Option C Tertiary structure alterations are exemplified by Duchenne muscular dystrophy that produces
dystrophin molecules lacking one or more domains. Prions alter secondary structure.
Option D Quaternary structur
tetramers) . Prions don’t have quaternary structure.
Incorrect Responses
Option A (bleeding gums) is incorrect. Vitamin C deficiency produces bleeding gums, but not dermatitis
and hair loss seen in biotin deficiency.
Option B (dementia) is incorrect. Niacin deficiency produces dermatitis as part of the pellagra syndrome,
but not hair loss seen in biotin deficiency.
Option C (glossitis) is incorrect. Riboflavin deficiency produces glossitis, but not dermatitis and hair loss
seen in biotin deficiency.
Option E (nystagmus) is incorrect. Thiamine deficiency produces Wernicke-Korsakoff syndrome with
symptoms like confusion, ataxia, nystagmus, but not dermatitis and hair loss seen in biotin deficiency.